You are on page 1of 178

Las Perlas de Infecciosas de Gompf

*ANTIBITICOS*

*Cuatro consideraciones al elegir un antibitico: ** * * TAEC* *T* toxicidad *A* actividad *E* espectro *C* Costo

**

*Causas de insuficiencia renal aguda relacionada a los antibiticos: ** * Aminoglucsidos = necrosis tubular aguda, (NTA), la insuficiencia renal no oligrica Anfotericina B-acidosis tubular renal, necrosis medular microisquemica, hipocalcemia, hipomagnesemia Aciclovir / sulfas obstruccin intratubular por cristales de hidrato los pacientes primero Nafcilina / oxacilina nefritis intersticial aguda Trimetoprim-artefactos, excrecin de creatinina reducida elevacin de la -Cr / K + alta

Vancomicina insuficiencia renal no oligrica (poco frecuentes menos que los niveles pico> 50) Colistimetato (colistina) insuficiencia renal oligrica

*Un paciente en la unidad de recuperacin del postoperatorio es difcil la extubacin.Cual**antibitico es ms adecuado para cubrirlo? ***

Gentamicina / aminoglucsido (puede causar bloqueo neuromuscular ya sea solo o en combinacin con agentes bloqueantes neuromusculares como succinil colina dar Ca + + IV para revertir).

Colistimetato / colistina, eritromicina y clindamicina pueden hacer lo mismo.

Podria exacerbar la miastenia gravis usar con precaucin.

*Su paciente est recibiendo daptomicina para osteomielitis SAMR, actualmente en la semana 4, as como el fluconazol para la candidiasis que comenz la semana pasada. l viene a la clnica quejndose de dificultad respiratoria y fiebre en los ltimos 2 das. Qu efectos adversos podra estar teniendo, y que ser responsable?***

La daptomicina esta asociada con una neumona eosinoflica que puede desarrollarse en dos o ms semanas durante la terapia. Los sntomas incluyen disnea, fiebre e infiltrados pulmonares que tienen una apariencia difusa como el edema pulmonar. El diagnstico puede hacerse mediante la bsqueda de IgE srica elevada, o eosinfilos en el lquido de los bronquiolos alveolares, biopsia pulmonar, e incluso derrame pleural. El tratamiento es la interrupcin de la daptomicina y una dosis progresiva de esteroides. La fibrosis pulmonar podria desarrollarse, pero el sndrome es generalmente reversible.

*Cundo se utiliza clindamicina vs metronidazol en la cobertura de las infecciones anaerobias? ** * La vieja regla de oro que clinda = por encima del diafragma y metro = debajo del diafragma, sigue siendo vlido. Clindamicina cubre tanto Gram + anaerobios, asi como Peptostreptococcus, Fusobacterium, Prevotella, Actinomyces, especies de Clostridium y otras de C. difficile, as como Gram anaerobios como Bacteroides spp. Metronidazol cubre anaerobios Gram asi como B. fragilis y todos los Clostridium, no deben utilizarse como monoterapia en la neumona por aspiracin la tasa de fracaso de un 50 por ciento y no en las infecciones graves en cabeza y cuello. Cubrir los anaerobios intestinales.

*BACTERIAS*

*Cul es el grupo HACEK? ** **

Bacterias Gram negativas las cuales son causas infrecuentes de infeccin intravascular y son difciles de cultivar (mantener CXS dos semanas).

*H* Hemophilus spp *A* Actinobacillus acetomycetamcomitans *C *Cardiobacterium *E* Eikenella *K* Kingella y *B*rucella, *C*oxiella burnetti (fiebre Q)

*Cules agentes patgenos humanos estn en los seres humanos como nico reservorio existente fuera del laboratorio? ***

Mycobacterium tuberculosis La poliomielitis La malaria La sfilis

Gonorrea El meningococo VIH Virus varicela zster El herpes simple El sarampin Paperas (Y antes de que fuera erradicada en el medio silvestre, la viruela)

[De ello se deduce que Adn y Eva fueron infectados con todos estos agentes y sobrevivieron, junto con sus hijos (quienes podran haber nacido con sfilis congnita y VIH, por lo menos). Suponiendo, por supuesto, que el Gnesis es de hecho en que todos los organismos vivientes fueron creados al mismo tiempo, y existen en forma inalterada.]* *

**

*Cules son los organismos SPICE / Spacek y por qu son importantes? ** *

*SPACEK * *S*erratia *P*seudomonas / Proteus indol + *A*cinetobacter *C*itrobacter *E*nterobacter *K*lebsiella Estos organismos pueden todos demostrar resistencia a betalactmicos y

puede requerir tto con carbapenem. Los organismos SPACE pueden producir un espectrum beta lacamasa de base amplia- cromosoma inducible como parte del grupo de Enterobacteracea, y la resistencia / fracaso puede ser inducida durante el tratamiento con betalactmicos, a pesar de que inicialmente el test sea susceptible. La preferencia de tratamientos en infecciones graves es un carbapenen (Primaxin / Merrem).E. coli y Klebsiella son los ms comunes productores de beta lactamasas de espectro extendido (BLEE), la pantalla de los laboratorios tantos los aislamientos si CMI para ceftazidima es> / = 2 microg / ml. Slo recuerde que la mayora de las enterobacterias se debe sospechar de BLEE, y puede requerir tx con carbapenem.Recuerde que Klebsiella tambin tiene un componente (o caracterstica) beta lactamasa de base cromosmica que confiere resistencia a la ampicilina / ticarcilina, por lo que estas drogas nunca son una buena eleccin para esta bacteria.

**

*Cuales bacterias son tipicamente productoras de gas? ** * Anaerobios / clostridios E. coli Klebsiella

*En cuales organismos piensa usted en las infecciones trasmitidas por el agua? ** *

Yersinia (agua potable sin tratar) Aeromonas (traumatismos, picaduras de reptiles) Vibrio (agua salada) Pseudomonas (foliculitis de la tina) Legionella (aerosoles de las unidades de aire acondicionado, fuentes de agua estancada en las instituciones) Micobacterias atpicas (neumona debida a los aerosoles de baeras de hidromasaje y duchas) Streptococcus iniae (celulitis a partir de la manipulacin del pescado tilapia) Naegleria fowleri (lagos de agua dulce caliente) Acanthamoeba (solucin de lentes de contacto, lagos) Leptospirosis (viajes de caza, natacin en lagos y ros, Hawaii) Criptosporidiosis (agua potable sin tratar) Cercarias dermatitis (esquistosomas aviar / reaccin alrgica) Esquistosomiasis / bilharziasis (Puerto Rico y el Caribe)

*Nombre clnicamente miembros importantes de las enterobacterias. Cules son fermentadores de la lactosa? ** * *KEEP* K-Klebsiella E-E. Coli E Enterobacter P Proteus

Todos estos son fermentadores de la lactosa.

*El laboratorio de Micro te llama para decirte que un cultivo de sangre que usted pidi ayer es positiva y la mancha Gram revela un no fermentan la lactosa, oxidasa positiva, GNR. Qu patgeno debe asegurarse de que est cubriendo con sus antibiticos, a la espera final de la identificacin? ***

Pseudomonas.

*La meningitis neumoccica se asocia con que factores de riesgo? ** * La sinusitis, la otitis media Las fracturas de crneo fugas de LCR /post-neurociruga Implantes cocleares *Quin est en riesgo de enfermedad neumoccica invasiva (ENT, sistema nervioso central o de los pulmones)?*** Clulas falciformes Hipoesplenia Hipogammaglobulinemia Mieloma mltiple Diabticos Alcohlicos / cirrticos

*Las caractersticas de la enfermedad de Whipple: (Tropheryma whippelii) *

* ***

Todos los estadounidenses Active Mush Charmin Dolor abdominal Artritis Adenopata Malabsorcin Confusin

Tto: Inicialmente Ceftriaxona + estreptomicina, luego TMP / SMX x 1 AO. *Directrices generales para hacer frente a la bacteriemia por Staphylococcus aureus: ***

Hay un enfoque extrable? En caso negativo: 4-6 semanas antibiticos En caso afirmativo, hay un soplo (algunos agregar dispositivos protsicos y los injertos vasculares)?

S: 4-6 semanas antibiticos por va intravenosa No: 2 semanas antibiticos por va intravenosa

Trate de encontrar focos extrable y drenable: tomografa computarizada de abdamen / pelvis, Echo / Ecocardiografia transesofagica, catteres IV, prtesis / injertos vasculares.

*Cules son las caractersticas clsicas asociadas infecciones por Staphylococcus aureus? ** * Pus amarillo dorado (aureus = oro) Descamacin de palmas de manos y plantas de pies Shock txico

**

*Cules organismos Gram + son resistentes a la vancomicina? ***

Leuconostoc Lactobacillus Propionobacterium Pediococcus Clostridia No difficile Erysipelothrix

Y, por supuesto, VREnterococcus, VRStaphylococcus aureus

*Que es lo caracterstico de las manifestaciones de la micobacterias tuberculosis ? ***

Presentaciones crnicas Abscesos fros / pus sin dolor (monoctica, inflamacin granulomatosa, no PMN) Piuria estril (TB)

*Qu es la BCG y por qu es importante?***

BCG (Bacilo de Calmette-Guerin) bacilo se utiliza para inmunizar a los lactantes, nios pequeos en pases en desarrollo contra la meningitis tuberculosa. No protege contra la infeccin de la TB y la infeccin latente de tuberculosis se producir independientemente de la condicin de BCG. Pero para 5-10 aos despus de la vacunacin, el PPD es + y hay mucha confusin sobre lo que significa. Antes de la vacunacin con BCG no causa un PPD > de 20 mm, que es la tuberculosis latente, es muy probable. A + PPD en el marco de la vacunacin con BCG antes deben ser tratados como si el paciente nunca tuvo una BCG (es decir, el estado de la BCG es irrelevante) porque no se puede descartar la infeccin tuberculosa latente.

* *

*Cules son las manifestaciones de la tuberculosis extrapulmonar? ** * SNC meningitis (meningitis basilar .parlisis Nerviosa Central, proceso crnico) ENT larngeo (altamente contagiosa!) otitis crnica / no dolorosa adenopata: cervical / SC / adenitis axilar (escrfula) mediastinitis fibrosante / sndrome de la SVC (imita histoplasmosis) pericarditis constrictiva GI enteritis peritonitis adenitis mesentrica GU piuria estril / renal prostatitis uterina sea osteomielitis / artritis sptica osteoporosis vertebral (Mal de Pott) La piel eritema indurado (Bazin enfermedad posterior de la pierna) prosector de verrugas

*Cuntos organismos deben estar presentes en una muestra de esputo para la baciloscopia para ser +? ** *

10.000 / esputo cc

*Cuntos micobacterium tuberculosis debe ser inhalado para que ocurra la infeccin? ** * 10-100**

**

***En un paciente con frotis de esputo negativos AFB cuya AFB culturas posteriores crecen la tuberculosis es contagiosa. ** Verdadero o Falso? *

Falso. Tres consecutivos negativos BK generalmente se acepta como una indicacin de que un paciente no es suficiente tos con AFB a ser infeccioso y puede salir del aislamiento.

* *

*VIRUS*

*Qu vacunacin infantil (que no sea la varicela) es importante

actualizar (dar una dosis de refuerzo) en adultos?*

Sarampin, paperas, rubola, especialmente en los adultos nacidos despus de 1957 o que hayan recibido una vacuna asesina ineficaz entre 1963 y 1967 y nunca lleg otro refuerzo. Este ltimo grupo podra desarrollar una infeccin de sarampin atpico.

Adultos nacidos antes de 1957 se supone que han estado expuestos al sarampin, y tienen inmunidad a largo plazo.

Mujeres en edad frtil, trabajadores de la salud, y viajeros quienes no tienen prueba de vacunacin despus de 1967 deben recibir 2 dosis de SPR.

Tambin actualizar ttanos-difteria cada 10 aos. **

*Qu es el sarampin atpico?***

Un sndrome de poliserositis de hipersensibilidad como resultado de la formacin de anticuerpos contra el sarampin de no proteccin.

*Dos enfermedades asociadas con infecciones por HTLV-1:*

Linfoma de clulas T del adulto/ leucemia Paraparesia espstica tropical

*P**rueba de laboratorio para diagnosticar las infecciones por HTLV-1: ***

HTLV-1 Ab Western blot con RIPA

*Cmo se transmite el HTLV-1 y donde es endmico? ** * Es un retrovirus como el VIH: Productos sanguneos Contacto sexual Transmisin vertical (la leche materna)

En el Sudeste Asitico.

*Manifestaciones de la paraparesia espstica tropical:*

Debilidad de las extremidades inferiores Ataxia Disfuncin vesical Espasticidad de las extremidades inferiores Incremento de los reflejos de la rodilla y del tobillo *Cules son las 6 enfermedades que el adenovirus produce?*

Fiebre faringoconjuntival (piscinas)

Conjuntivitis epidemica / queratitis (piscinas) Enfermedad respiratoria aguda (grave, epidemia de campo URI de inicio) Cistitis hemorrgica aguda (nios menores de 15 anos, autolimitada) Gastroenteritis /? asociados con invaginacin intestinal Infeccin adenoviral / fiebre de origen desconocido en pacientes trasplantados *Qu hallazgo fsico es patognomnica de conjuntivitis por adenovirus, fiebre faringoconjuntival *

Linfadenopata preauricular (Gonoccica, conjuntivitis por clamidia produce esto tambin, pero menos frecuente y esta asociada con la actividad sexual)** **

*Qu virus producen cistitis hemorrgica?*

Adenovirus (ms comn en nios sanos, autolimitada) virus BK (post-trasplante) **

*Cules son las 6 enfermedades que el Parvovirus B-19 causa? ** * El eritema infeccioso / La quinta enfermedad Aborto, hidropesa fetal Infeccin crnica en los inmunodeprimidos Artropata (manifestacin ms comn en adultos) Anemia falciformes Posible fatiga crnica y fiebre de origen desconocido (PCR en sangre)

Dg: IgM, 4x aumento de la IgG elaborado dos semanas aparte; pro-normoblastos gigantes en la biopsia de mdula sea

*Qu enfermedades causa el HHV-6? ** * Exantema sbito o rosola infantum (Fiebre alta en un nio generalmente cmodo, seguido por desaparicin de la fiebre y erupcin cutnea). Fiebre en pacientes post transplantados de medula osea.

*Qu causa el virus de JC?*

Leucoencefalopata multifocal progresiva en SIDA (Demencia progresiva, declinacion neurolgica, muerte dentro de 1 ao)

* *

*Cuals procesos de enfermedad estn asociado con el virus de Epstein-Barr?*

Mononucleosis infecciosa aguda (infeccin primaria por EBV; fiebre, faringitis exudativa, esplenomegalia, linfoadenopatas, hepatitis,

fatiga profunda que puede ser el sntoma ms destacado en los adultos, se resuelve en varias semanas) Sndrome linfoproliferativo Ligada al cromosoma X (mono fatal en nios genticamente predispuestos)

Leucoplasia vellosa oral (en SIDA)

Muy rara: infeccin Crnica del VEB (por lo general inmunodeprimidos) [VEB no causa el sndrome de fatiga crnica.] **

*Qu tumores malignos se asocian con el VEB?*

Linfomas Linfoblstica de clulas B Linfoma de Burkitt Linfoma de clulas B relacionadas con el SIDA Carcinoma nasofarngeo Linfomas postrasplante (especialmente despus de las terapias antilinfoctica OKT3) Algunos linfomas de clulas T ? linfoma de Hodgkin

Es el aciclovir u otros antivirales tiles en las enfermedades asociadas al VEB?

NO. Debido a manifestaciones de la enfermedad de VEB, incluidos mono aguda, se relacionan con la activacin inmune (activacin de linfocitos B y T). Al tiempo de comenzar los sntomas, la mayor parte de la

replicacin viral se haya resuelto.

* *

*HONGOS*

**Qu* organismos fngicos se asocian con los estados de sobrecarga de hierro y el uso desferoxamina?*

Mucormicosis (Rhizopus) w-TAMBIN Asoc. con diabetes

*Describir la apariencia microscpica del Blastomyces (Blastomicosis) y el nombre de **4 rganos y sistemas que afecta con mayor frecuencia.*

Micro formas incipientes de base amplia de la levadura en muestras de tejido. Pulmones Piel GU- prstata, epiddimo, testculos Hueso

**

***Describir la apariencia microscpica de coccidioidomicosis.*

Esfrulas que contienen muchas esporas en el tejido. Forma de barril cuentas en las cadenas filamentosas en el cultivo.

*Describir la apariencia microscpica de histoplasmosis.*

Racimos agrupados de levaduras que aparecen nucleadas, a menudo en un PMN, en el tejido.

**

***Describir la apariencia microscpica de Cryptococcus. ***

Levaduras individuales, ocasionales brotes apretados, con una cpsula de grasa, en frotis de tejido lquido

*Describir la apariencia microscpica del Sporothrix.*

Con forma de cigarro levaduras en los tejidos.

*Describir la apariencia microscpica del Paracoccidiodes.*

Levadura grande con brotes mltiples de una clula central (rueda

marinero) en el tejido.

*Describir la apariencia microscpica del Aspergillus.*

Aguda de ramificacin (ngulo de 45 grados) hifas septadas en el tejido.

*Describir la apariencia microscpica de Mucor*

Amplia asepstate hifas ramificadas en ngulo de 90 grados en el tejido. **

*Describir la apariencia microscpica del phaeohyphomycoses (hongos dematiceos producen lesiones negras.)*

Las hifas que se parecen a los granos de grasa ensartados, o buscar escasos a intervalos en el tejido.

* *****

**Qu* es lo caracterstico de la endocarditis por hongos?*

Vegetaciones grandes

Embolia arterial extensa (por ejemplo, pies fros sin pulso en un Drogas de uso IV)

Endocarditis fungica es una indicacin absoluta para el reemplazo de la vlvula. Tratar con anfo B IV y ciruga lo antes posible.

*Quin est en riesgo de endocarditis por hongos? * Drogadictos abuso de drogas intravenosas (20% de los UDIV endocarditis por hongos) Inmunosupresin Post ciruga cardaca

*Qu hongos estn asociados con endocarditis en que los individuos? ** * Drogadictos (uso EV) -Candida parapsilosis Inmunosupresin = Aspergillus Post-ciruga cardaca = Candida spp. **

*Cules son las complicaciones ms graves de la bacteriemia por Candida? ** * Endoftalmitis (ver por la prdida visual, al examinar la retina bolas

de algodn, consulte Oftalmologa STAT) Endocarditis / embolia

*Qu se debe sospechar en un paciente con SIDA y una historia de viajes a la mitad occidental del valle del ro Ohio y el valle del ro Mississippi y cmo se diagnostica? ***

La histoplasmosis. Las pistas: Linfadenopata Hepatoesplenomegalia lceras orales Txicos, sptico Tambin los viajes hacia el Caribe, Sudamrica Diagnstico: antgeno de Histoplasma urinaria o en suero Tratamiento: IV ampho B durante 2 semanas despus itraconazol PO Nota: en huspedes normales, Histo tambin puede producir enfermedad cavitaria indolente cronica, ndulos pulmonares.

***Cules** hongos causan las micosis sistmicas verdaderas (no necesariamente oportunistas), cul es su preferencia o asociacin geogrfica en EE.UU, y Drogas de eleccin? ** *

Histoplasmosis: guano de murcilago- / cuevas / palomas; valles de los ros ro Ohio y Mississippi -Ampho B Coccidioidomycosis- suelo/SO USA-fluconazole/itraconazole, Ampho B Blastomicosis suelo-hmedo? / Medio Oeste estadounidense / SE USA itraconazol Paracoccidioidomicosis-suelo? / Por lo general en Amrica Latina (blastomicosis sudamericana), itraconazol

*Qu causa la infeccin por hongos en pacientes que son diabticos o tienen sobrecarga de hierro? ** * Mucor / especie Rhizopus. Si el nivel de ferritina es alta, sin otra explicacin (recuerde que la ferritina es un reactante de fase aguda), esto puede indicar un riesgo para estos hongos.

* *

*PARSITOS*

*La malaria es una enfermedad FAST. Nombre de sus 4 caractersticas.*

F Fiebre A- Anemia

S Esplenomegalia T Viajes en el ltimo ao a la zona endmica

fiebre Negro agua se refiere a negro / orina de color oscuro que ocurren durante los perodos de hemlisis de P. falciparum, especialmente con la quinina / quinidina.

**

*Cul Plasmodium es el peor para tener? ** * P. falciparum causa malaria maligna, gametocitos en forma de banana; parasita > 4% de los eritrocitos (eso es mucho!). > 1 trofozoto por campo de inmersin en aceite en una gota gruesa sugiere 10% de parasitemia, eso es por P. falciparum! Multiplicacion parasitaria en los glbulos rojos, eso es por P. falciparum! Tropismo perifrico o superficial P. falciparum! -No puntos P de Schuffner. falciparum!

TX: cloroquina o mefloquina (por lo general por P. falciparum es resistente a la cloroquina, excepto en Hait); atovacuona / proguanil (Malarone) tambin est aprobado para la profilaxis o tratamiento de P. falciparum.

*Cmo causa la muerte el P. falciparum?***

Hiperparasitemia (> 250.000 eritrocitos parasitados / microlitro de sangre en una gota gruesa) se produce, especialmente en los viajeros que nunca han sido expuestos a la malaria. Las personas que viven en las zonas endmicas son semi-inmune y menos propensos a tener hiperparasitemia / complicaciones graves.

Esto, junto con la rigidez inherente / la aglutinacionde los trofos, causa espesamiento en las arteriolas / capilares / hemlisis masiva isquemia cerebral difusa es la ms peligrosa para la vida; seguido por la anemia severa, hipoglicemia, acidosis lctica (trofos usan la gluclisis anaerbica , que produce lactato, derivados de la quinina estimulan los islotes productores de insulina tambin), insuficiencia renal, hipoxia, tambin tienen diarrea, edema pulmonar tardio, isquemia cardiaca, etc

*Cules son las complicaciones de P. vivax / oval? ** * *Anemia *severa. Estos tropos no pegajosos y slo parasitan los glbulos rojos jvenes, por lo que no producen las dems complicaciones de P. falciparum, se reproducen cada 48 horas (por lo que la fiebre se produce cada 48 horas como progreso de la enfermedad, y se reproducen a un ritmo menor) .

*Ruptura del Bazo* + 2.3 meses despus de la resolucin (incluso con la palpacin en el examen abdominal).

*Cules son las complicaciones de P. malariae?*

*Glomerulonefritis* inmune compleja. Esta es una parasitemia de bajo nivel con pocas complicaciones agudas que no pueden ser recogidos por muchos aos. Los complejos inmunes son los anticuerpos anti-parsito y antgenos de P. malariae. *Qu grupo de personas semi-inmune se encuentra en un riesgo similar de complicaciones como individuos no inmunes?*

Primagravidas.**

*Cmo estimar el nivel de parasitemia (densidad parasitaria)?*

Densidad parasitaria por microL = [Cuenta # parasitos/200 GB en frotis] X [Total de glbulos blancos del CTB/200] Estimar hacerlo en una gota gruesa de sangre, el cual es visto bajo aceite de inmersin. Un frotis de sangre debe extenderse justo al espesor adecuado para leer el peridico a travs de l.

P. falciparum => parsitos 250KParasitos /microL Parasita los glbulos

rojos de todas las edades P. vivax / oval = <50K/microL Parasita los glbulos rojos ms jvenes P. malariae = <10K/microL Parasita los glbulos rojos mas viejos

En general,> 1 trofo por campo de inmersin en aceite = P. falciparum.//

*Cmo ms se puede decir la plasmodios aparte? ** * P. falciparum- gametocitos en forma de pltano P. vivax / oval de color rojo puntos de Schuffner en los eritrocitos P. malariae- gametocitos como banda- que se extienden a travs de los eritrocitos

*Cmo funciona el rasgo de clulas falciformes proteccin contra P. falciparum?*

La malaria en el continente africano es la razn predominante de que el rasgo de clulas falciformes se ha mantenido en los seres humanos, como una ventaja evolutiva. Los glbulos rojos parasitados son secuestradas en la circulacin perifrica y la formacin de clulas falciforme produce poco oxgeno inhibiendo el crecimiento de P. falciparum.

*Cul de los **Plasmodios pueden recaer hasta 5 aos despus de la infeccin y por qu?*

P. vivax / ovale pueden manifestarse el paludismo varios meses despus de la infeccin inicial, debido a un ciclo persistente en el higado (los hypnozoite en estado latente no se replican) despus de un tratamiento inadecuado en la fase inicial. La primaquina se aade a otras terapias para curar la fase heptica.

P. malariae puede persistir por hasta 30 aos a pesar del tratamiento**

**

*Cul es el vector, las manifestaciones, pruebas diagnsticas y el tratamiento de la leishmaniasis?*

Flebotominas flebtomos (Texas, S. de Asia / el Medio Oriente / Amrica Latina)

La leishmaniasis cutnea planteadas, como la pizza (base roja, exudados amarillo) lesiones de destruccin de la cara central (espundia, Amrica Latina, L. brasilensis), biopsia de los borde de amastigotes.

La leishmaniasis visceral (kala azar) perodo de incubacin 3-8 meses, fiebre o hepatoesplenomegalia masiva / debilidad, biopsia del hgado / bazo / MB por amastigotes

Tratamiento: Ampho B para el kala azar; cutnea puede curar, pero si hay afectacin de las mucosas o la cara, el estibogluconato /antimonio pentavalente IV

**

*Qu es la enfermedad de Chaga, el vector, pruebas diagnsticas y tratamiento?*

La enfermedad de Chaga (Trypanosoma cruzi): tripomastigotes en forma de C en la sangre -Aguda: signo de Romaa (edema periorbital), fiebre, miocarditis -Crnicamente: fiebre, hepatoesplenomegalia, acalasia / megacolon, miocardiopata

Vector: redvidos (triatomas) chinches (Amrica Latina y el de Texas) Diagnstico: aguda (frotis de sangre para tripomastigotes), crnica (ELISA) DDE: nifurtimox, benznidazol

*Cules son las dos formas de la enfermedad africana del sueo, el vector, diagnsticos y tratamiento?***

*Trypanosoma brucei rhodesiense (forma del frica Oriental)-EMERGENCIA *das o semanas: fiebre alta repentina, mialgia, somnolencia / HA / corea, chancro tipo bocado indoloro, coma / muerte con/sin tx inmediato, invrementado en viajeros / vacaciones de safari

*T. b. gambiense (forma de frica Occidental)* -indolente/semanas meses, somnolencia / corea / como Parkinson, ganglios cervicales

posteriores prominentes (signo de Winterbottom), hepatoesplenomegalia * Vector: mosca Tse tse (African Safari) Diagnstico: gota gruesa y fina de tripomastigotes *DDE: suramina, pentamidina (agentes de arsnico: melarsoprol / tryparsamide si el SNC recibe de los CDC)****

*Qu**es importante acerca de las infecciones por Strongyloides?*

S. stercoralis puede persistir en el husped durante dcadas a travs de un ciclo de autoinfeccin donde las larvas que eclosionan en el intestino puede volver a entrar en el torrente sanguneo, los pulmones, son expulsados, y se ingiere, madura en el tracto GI, ponen huevos, las larvas eclosionan y repite el ciclo; curiosamente, la larva tambin pueden ser excretados y tienen un ciclo de vida totalmente independiente en el suelo.

En el husped normal, la infeccin intestinal por lo general asintomticos con eosinofilia perifrica. Si la carga pesada, sibilancias, neumona (sndrome de Loeffler) puede producir, as como diarrea, mala absorcin, urticaria. *En pacientes inmunodeprimidos, el sndrome de hiperinfeccin se produce*:

- infiltrados pulmones difusos -Dolor abdominal -Meningitis

- Sepsis por Gram negativos (a partir de la penetracin intestinal de larvas) -NO eosinfilos

Dx: Piense en esto en el paciente que est inmunodeprimido, tiene caractersticas de lo anterior, y ha vivido en el sur rural o en los trpicos. En el sndrome de hiperinfeccin, el organismo es encontrado en sangre, LCR, esputo / BAL, orina. De lo contrario, las heces de O y P X 3 o prueba de heces.

Tx: tiabendazol x 2 d (2-3 semanas si hiperinfeccin). El mejor tratamiento para la hiperinfeccin es evitarlo mediante el tratamiento antes de inmunosupresin

*Describir las tres principales nematodos adquiridas por ingestin oral-fecal: ** * *Ascaris:* (un gusano redondo de cerdo de 1 pie de largo), Estados Unidos rural / SE, malabsorcin / esteatorrea, le gusta obstruir la va biliar / intestino delgado, fcil de ver en O y P de huevo grande, con pelo spero; mebendazol x 3d

*Trichuris:* (triquina), rural SE Unidos y Puerto Rico, deficiencia de hierro, diarrea con sangre / prolapso rectal; O & P en forma de un baln de futbol americano con extremos conectados; mebendazol x 3d

*Enterobius:* (oxiuros), comn en todas las clases sociales y / o nios, muy resistente en el medio ambiente / hojas / polvo, prurito perianal nocturnos, pesadillas, utilice cinta de teipe en el rea perianal donde los gusanos filiforme ponen los huevos en la noche, el tratamiento de toda la familia con mebendazol una dosis semanal x 2

*Cules son los anquilostomas y qu causan?*

Necator americanus y Anquilostoma duodenale. Penetran por la piel de los pies (comezn de tierra), entrar en los pulmones / trquea, se tragan, se adhieren a intestino delgado, donde chupan (mucho) la sangre y ponen huevos.

Principales causas de la deficiencia de hierro en todo el mundo. Fcil diagnstico por heces huevos y parasitos. Tx con mebendazol x 3d.

*Qu gusano / nematodo causa edema periorbitario y mialgias?*

Trichinella spiralis, se ingiere como un quiste (de color rosado) de carne mal cocida de cerdo, oso, morsa, puma.

diarrea, vmitos, dolor abdominal, a continuacin, mialgia orbital, edema periorbitario, conjuntivitis mialgias, debilidad muscular prolongada miocarditis

eosinofilia / Creatinina PhosphoKinasa alta / baja Velocidad de Sedimentacion Globular

Tto con tiabendazol por 1 semana de la exposicin (mata a los gusanos intestinales, no en el msculo), si no de apoyo.

*Cules son los cestodos / tenia una gran importancia clnica y por qu?* Echinococcus (equinococosis heptica); perros pastores en el suroeste de EE.UU. y en todo el mundo, la reseccin quirrgica (no se derrame el contenido del quiste anafilaxia; en el hgado, el contenido del quiste puede ser aspirado e inyectar etanol para matar a vesculas hijas)

Taenia solium (neurocisticercosis), lesiones del SNC / convulsiones / paraplejia; dx por RM / serologa; tto con / reseccin quirrgica, esteroides + / prazicuantel, antiepilpticos

*Cules son las aletas / trematodos de mayor importancia clnica? ***

Esquistosomiasis Aviar, la picazn del nadador en Grandes Lagos, autolimitada

Schistosoma (spp mansoni / haematobium / japonicum). Cercaria en el agua entran a travs de la piel, sangre / hgado y las larvas migran a los pulmones.

Intestino delgado / venas mesentricas superiores (S. japonicum) Intestino grueso / venas mesentricas inferiores (S. mansoni) bilharzia + Eosinofilia, fiebre, huevos en heces, hepatoesplenomegalia, hipertensin portal no cirrtica Biopsia de tejido, serologa Vejiga / plexo venoso vesical (S. haematobium); + Eosinofilia, fiebre, orina / huevos pared de la vejiga, hematuria, hidronefrosis, infecciones del tracto urinario, eyaculacin dolorosa, CA vesical Tto con prazicuantel x 1d. *FIEBRE Y SEPSIS*

*Mnemotcnica del Dr. Sinnott para la formulacin de un diagnstico diferencial en las fiebres (o casi nada en la medicina!): ** * Congnita Infeccin Neoplsicas Endocrinas Metablica Autoinmune Txica Vascular *Lista 8 fuentes comunes de la fiebre en un paciente en la UCI: ** * Lneas Pulmn (infeccin, atelectasia) Heridas

Orina (Tambin conocido como la herida, Agua, Viento, Paseo) Senos paranasales Prstata Candidemia Drogas **

*4 causas no infecciosas de la fiebre en un paciente en la UCI: ** * *IDEA*

*I *Infarto al Miocardio

*D *Drogas

*E* Embolismo Pulmonar

*A *Addison

*En el contexto de la fiebre, Qu pistas anuncian Peligro!?** ** * Petequias / purpura pensar en meningoccemia, nunca golpe que fuera! Cefalea pensar en meningitis Viajero pensar en paludismo, tripanosomiasis africana oriental (safaris al parque)

Rigors pensar bacteriemia / sepsis Asplenia pensar sepsis postesplecnectomia tras la abrumadora-JUMP en esto! Hipogammaglobulinemia r / o sepsis

*Cul es el diagnstico diferencial de petequias?*

*Trastorno de coaglulation* **

*Trastorno plaquetario* (por ejemplo, TTP. ITP, quimio-relacionados) **

*Fiebre de las Montaas Rocosas* (23% de mortalidad si el tto se retrasa 5 das; otras rickettsias pueden causar petequias, tambin)

*Meningococcemia / meningitis neumoccica* (alta mortalidad)

*Endocarditis* (aguda / estafilococo dorado rpidamente progresiva puede ser mortal) Eliminacin de grasas y mbolos de colesterol *Las causas ms comunes de fiebre en los pacientes quemados:** *

Condritis supurativa Parotiditis supurativa Prostatitis Flebitis

*Pirgenos endgenos son factores solubles que provocan fiebres.** **Nombre 4 de esos** **pirgenos.** * Caquectina (factor de necrosis tumoral) Interleucina-1 Interfern-alfa MIP (protena inflamatoria del macrfago) Prostaglandinas se utilizan como mediadores secundarios por estos pirgenos y puede ser inhibida por los inhibidores de PG.

*Definicin de Fiebre de origen desconocido (FOD).** **Lista de causas comunes con frecuencia** **de ocurrencia.*

Fiebre de origen desconocido = T 101 para 3 semanas sin diagnstico despus de la evaluacin una semana intensa.

Infeccin (especialmente la tuberculosis) 30% Neoplasia (especialmente leucemias y linfomas) 20% Misc. (Fiebres de drogas, Embolismo Pulonar, enfermedad inflamatoria intestinal), artritis temporal, Polimialgia reumtica, enfermedad vascular del colgeno) 20% Sin diagnosticar 15%

*Causas de una bradicardia relativa (dficit de pulso-temperatura o

signo de Faget: fracaso en el incremento del pulso apropiadamente en el contexto de la fiebre -10 ppm / 1? F por encima de 98.6 F?):** *** /Clsicamente:// // /Fiebre tifoidea (fiebre entrica, causada por Salmonella) Legionelosis Brucelosis Leptospirosis Psitacosis Fiebre por Drogas

Tambin /alteraciones de la conduccin/ con: Beta bloqueadores Fiebre reumtica aguda Enfermedad de Lyme Miocarditis viral Endocarditis infecciosa //

//

*Causas comunes y vecinas de la fiebre de drogas** ** * Anticonvulsivantes (Dilantin, Tegretol) Minociclina Otros antibiticos (betalactmicos, sulfamidas y nitrofurantona) Alopurinol Colace Heparina

*Cul es el diagnstico diferencial de Sinnott del sndrome de la sepsis? * **

*T*aponamiento *S*DRA *I*nfarto al Miocardo *E*mbolismo Pulmonar *S*ndrome compartimental abdominal

**

*Cuando ves el sndrome compartimental abdominal y por qu?*

Aumento de la presin en un espacio cerrado amenaza la viabilidad de los tejidos circundantes y causa la disfuncin del rgano. El no reconocer la presencia de hipertensin intraabdominal antes que ACS se desarrolle conduce a hipoperfusin, falla orgnica multisistmica, y las tasas de mortalidad de 40 100%. Presin capilar pulmonar capilar y aumento de la presin venosa central con el aumento de la presin intraabdominal (PIA), a pesar de retorno venoso y reduce el gasto cardaco.El sndrome de compartimiento abdominal se observa con:

Reposicin de volumen masivo Obstruccin intestinal

Pancreatitis Ascitis masiva Peritonitis Sangre Intraperitoneal Distensin Intestinal o tercer espacio de lquidos

*Cmo se mide PIA?** ** * 50 ml de solucin salina estril al se instila en la vejiga a travs del puerto de aspiracin de una sonda de Foley con el tubo de drenaje cerrado. Una aguja de calibre 18 unida a un transductor de presin se inserta en el orificio de aspiracin, y la presin se mide. ACS no est con una presin inferior a 10 mmHg y por lo general se presentan con una presin de 25 mmHg.

*Cules son los grandes imitador enfermedades de las enfermedades infecciosas?** ** * TB

Sfilis VIH

*Cules son las causas de meningitis asptica y sus claves?** *

Frecuentes: VIH (infeccin temprana, antes de la carga del VIH comprobar ab + viral o antgeno p24; la prueba del VIH) Enterovirus (verano / otoo) VHS (recurrente = Mollarets) Meningitis bacterianas parcialmente tratadas (antes de VO / ABT IV)

Menos frecuentes: Varicela (enfermedad activa) TB (exposicin a la tuberculosis / PPD+) Brucella (cabras / mamferos de pezua) Virus de la coriomeningitis linfoctica (hmsters) Sfilis (enfermedades de transmisin sexual) Lyme / ehrlichiosis humana monoctica (garrapatas) **

**

*Cules son las causas de meningitis asptica recurrente?** *

La meningitis es Mollaret (clsico, causados por el VHS) Sndrome de Vogt-Koyanagi-Harada (meningitis asptica, uvetis, dficit del 8vo nervio craneal) AINE (especialmente en mujeres jvenes con lupus)

Sndrome de Behcet

*Una lesin necrtica / negra puede ser visto en:** ** * Infecciones por moho Ectima gangrenosa (sepsis Pseudomonas en neutropenicos) Antrax cutneo (rodeada por edema gelatinoso) Rickettsia conorii (tache noire) Meningococcemia / sepsis neumoccica grave Herptica / herpes zoster mbolos de colesterol

*Qu organismos se sospecha que con una faringitis exudativa?** ** * Estreptococos del grupo A (muy comn) Virus del Epstein Barr (mononucleosis infecciosa) Grp A estreptococo pueden co-infectar con mono Difteria (exudados crecen juntos en una membrana que debe ser eliminada) llamar al departamento de salud si se sospecha!

*Qu organismos deben motivar la bsqueda de una neoplasia GI subyacente?** *

Streptococcus bovis (bacteriemia / endocarditis) Clostridium septicum (celulitis crepitantes sin un punto de entrada)

*Principios generales de utilizacin de las vacunas e IG:*

* *Las IG y las vacunas no vivos no deben administrarse conjuntamente en el mismo sitio o una jeringa porque el IG interfiere con el montaje de la respuesta de anticuerpos. La vacuna de la rabia y RIG, la vacuna contra el ttanos y TIG, vacuna contra la hepatitis B y HBIG, la vacuna contra la viruela y VIG se dan al mismo tiempo posterior a la exposicin Las IG y vacunas vivas no se deben administrar junto a todos: MMR y la varicela se puede administrar ms de 3 meses despus de IG. IG se puede administrar 2-3 semanas despus de la triple vrica o la varicela. Las vacunas del clera y la fiebre amarilla deben darse con tres semanas de diferencia.

2 o> vacunas vivas deben darse simultneamente o 4 semanas de diferencia, excepto la polio oral, fiebre tifoidea oral, y la fiebre amarilla se puede dar en cualquier momento.

La IG Varicela hay post ya propuesta de la exposicin a menos que un individuo no es inmune y inmunodeprimidos o embarazadas. La vacuna se le ofreci despus del parto o si / cuando se resuelve la inmunosupresin.

IG (tambin conocido como inmunoglobulina srica, gammaglobulina) contiene las cantidades especficas de anticuerpos contra el sarampin, la difteria y la polio, cantidades variables de la hepatitis A y B, varicela, VRS, otros.

IG especficas incluyen: la hepatitis B IG, varicela zoster IG, IG rabia, el ttanos IG, y la vaccinia IG.

* *

*HUESOS*

*Lista de los 3 tipos bsicos de la osteomielitis, su etiologa bacteriana comn y la tasa de curacin:** * Hematgena Staph 90% de curaciones Contigua 50% gramnegativas cura y estafilococos (Staphylococcus aureus: un 50-60%) Neurovascular Anaerobios 10% de curaciones

*Lista 4 complicaciones de la osteomielitis:*

Amiloidosis Fstulas A-V Fracturas No soldadas (pseudoartrosis) CA de clulas escamosas en el tracto sinusal persistente ms de 20 aos Acortamiento del miembro

*Cules son las posibles complicaciones infecciosas de las fracturas (abierta) compuestas?*

Gangrena gaseosa (Clostridium perfringens esporas del suelo / contaminacin de la grava) Osteomielitis Aguda (y muchas veces crnica)

**

*Definir discitis, el patgeno ms probable, y DDE:*

Discitis = inflamacin o infeccin de un espacio en el disco intervertebral.

Patgeno = estafilococo. aureus en adultos y nios.

Tto = oxacilina / nafcilina.

*Lista de los hallazgos fsicos como un amplio absceso epidural:*

Localizacion del dolor / dolor a la percusin Debilidad distal / anestesia sensorial aberraciones en silla de montar Retencin urinaria / incontinencia intestinal, incontinencia por rebosamiento ms tarde Parlisis

**

*Qu organismos son encontrados en las reas del trauma del hueso?*

Tuberculosis y S. aureus**

* *

* *

*GASTROENTERIC**O*

*Lista 5 agentes bacterianos causantes de enfermedades gastrointestinales por la produccin de toxinas, y dar DDE****para cada uno.*

Campylobacter jejuni spp feto (eritromicina / quinolonas) Clostridium difficile (metronidazol) Shigella (quinolonas) Salmonella typhi (quinolonas / ceftriaxona) E. coli 0157: H7 (sin antibiticos, puede aumentar la produccin de toxinas y riesgo de sndrome urmico hemoltico!)

*Liste 3 patgenos invasores bacterianos y la DDE de cada uno.*

Yersinia enterocolitica (quinolonas / TMP-SMX) Vibrio parahemolyticus (quinolonas probablemente no cambiar el curso de la enfermedad) Vibrio vulnificans (celulitis) (doxiciclina y ceftazidima / cefepima)**

*Liste 3 hallazgos fsicos de la fiebre entrica, agente responsable, y la DDE para cada uno.*

Fiebre entrica con Salmonella typhi = Fiebre tifoidea. Tasa del Pulso ms baja de lo esperado para el grado de fiebre (signo de Faget) Manchas rosadas en el trax Esplenomegalia Agente: Salmonella typhi, con menos frecuencia S. paratyphi o S. choleraesuis

TX: quinolonas / ceftriaxona. En caso de shock, dar dexametasona x 8 dosis, comenzando antes de los antibiticos para disminuir la mortalidad.**

*Qu es la escombroidosis y cmo se diagnostica?*

Intoxicacin por histamina, que incluye enrojecimiento, picazn, urticaria, angioedema, sibilancias, vmitos, diarrea (reaccin anafilctica). Bacterias en la superficie de pescado inapropiadamente congelado degrada la histidina para histamina. Por lo general, atn, caballa, mahi bonita, y jurel, pero tambin otros. El pescado puede tener un sabor picante. 100 mgr de histamine/100g de pescado = escombroidosis (Por lo general, los hallazgos clnicos + historia de la ingesta de pescado son diagnsticos) Tratamiento con antihistamnicos y los bloqueadores de H2, adrenalina SC si es grave.

**

*Qu es la ciguatera?*

Neurotoxina relacionados con la ingestin de peces de arrecife, tales como la barracuda, el mero, el pargo, el gato. 1-6 horas despus de comer, dolor abdominal, diarrea, vmitos con sntomas neurolgicos caractersticos: Inversin de la sensacin de fro y calor Parestesias periorales

Parestesias, prurito intenso Vrtigo Dolor de cabeza Hipotensin Tx con manitol 1 g / kg IV, puede ser til, de lo contrario atencin de apoyo.

*Cules son las principales intoxicaciones neurotxicos por mariscos?*

Intoxicacin paraltica por mariscos Almejas, mejillones, mariscos Alaska, Maine, el Pacfico Noroeste Intoxicacin neurotxicos por mariscos brevetoxina / mareas rojas Florida, El Golfo, las Costas del Atlntico

**

*Qu microorganismos se asocian con el abuso crnico de alcohol o enfermedad heptica, y cules son las fuentes probables?*

Piensa hgado: LYVA Listeria (productos lcteos no pasteurizados, embutidos / perros calientes, carnes procesadas) Yersinia (de agua dulce, lacteos, carnes) Vibrio vulnificus (agua de mar, pescados y mariscos)

Aeromonas (agua dulce)**

*Nombre ocho causas virales de la hepatitis:*

La hepatitis B La hepatitis A La hepatitis C La hepatitis E (hepatitis fulminante especialmente en las mujeres embarazadas) Delta virus (la coinfeccin con hepatitis B es obligatorio) CMV Epstein Barr (VEB) Fiebre amarilla

**

*Cul es el significado de la hepatitis G (HGC o VHG) en el VIH?*

Se puede retrasar la progresin del VIH. Los datos de 2007 no demostrar la mejora de la mortalidad en la coinfeccin VHG-VIH.

*Cules son los factores de riesgo para la hepatitis E (VHE)?*

Viajes a zonas endmicas (frica, Afganistn)

Agua contaminada / falta de tratamiento de aguas No person-person/ transmisin sexual.

*Qu condicin predispone a graves VHE?***

El embarazo aumenta el riesgo de la hepatitis sintomtica ~ 10X (2 20%), especialmente hepatititis fulminante del tercer trimestre.

*Cul es el significado clnico de la mutacin promotor del ncleo (mutacin pre-nuclear) en la infeccin por el VHB?*

La infeccin crnica por el VHB suele producir HEAG, que se ha utilizado para el seguimiento de la respuesta al tratamiento. HEAG + tambin anuncia el aumento de la infectividad. Sin embargo, el VHB mutante pre-nuclear no puede producir HEAG HeAb -. Y mutantes del VHB es ms grave, ms progresista, y menos sensible a la terapia. Afortunadamente, los mutantes del VHB todava no es muy comn en los EE.UU., pero la posibilidad debe tenerse en cuenta para el paciente quien puede recaer despus del tto o tiene una enfermedad grave, a pesar de HEAG.

*Cules son las caractersticas de los pacientes con VHB mutante pre-nuclear?*

Residencia fuera de EE.UU. Asociados con genotipos distintos de A (genotipo del VHB es frecuente en EE.UU.) Adquisicin de larga tiempo Ms graves, hepatitis progresiva Baja respuesta al tto podr exigir la supresin de toda la vida

*Cmo la Hepatitis D (VHD) se diferencian en EE.UU. desde otros lugares?*

Ocurre sobre todo en UDVP, transfusin de sangre Raras aqu comn en el Amazonas

*Qu debe saber sobre la hepatitis C genotipo 4?*

Igual que el genotipo 1 para fines de tratamiento (no tratarlo como un no-1 genotipo) % muy bajo en EE.UU., pero ocurre Tenga en cuenta que muchos estudios de VHC en 1990 de tratamientos comparados entre los genotipo 1 frente a no-1, con no-1 supone que los genotipos 2 o 3. Esto se debe a que 4 no es tan comn en Occidente

como en otras partes, pero no dejes que te confundan los trminos. Para todos los propsitos prcticos, 1 = 4.

*De qu manera es VHC similar al VIH?*

Ambos son ARN virus Ambos se caracterizan por la variabilidad antignica, a pesar de producir anticuerpos, no es de proteccin. Miles de millones de anticuerpos se producen en respuesta a miles de millones de viriones producidos diariamente con capas de protenas diferentes. La produccin de anticuerpos simplemente no puede ponerse al da. El organismo los reconoce como cepas diferentes, o cuasiespecies. Esto es el por qu la produccin de la vacuna contra el VHC y el VIH ha fallado.

*Lista 5 causas bacterianas de la hepatitis:*

Leptospirosis Gonorrea Sfilis Salmonella Coxiella burnetii

*Protozoo que causa la hepatitis:*

Toxoplasmosis

**

*Lista de los 3 tipos de diarrea infecciosa, rea afectada, los agentes asociados.*

No inflamatoria: intestino delgado proximal, diarrea acuosa, sin PMN; Vibrio cholera, E. coli, Giardia Inflamatoria: colon; disentera; PMN +, Shigella, Vibrio parahemolyticus, C. difficile. Penetrante: intestino delgado distal, fiebre entrica; + / PMN, Yersinia enterocolitica, Salmonella typhi, Entamoeba histolytica Colitis por C. difficile: Causa = antibiticos (oral, IM, IV, tpico); recada 20%; anestesia; uremia; metotrexate. Clnica: profusa, diarrea acuosa, en la leucemia / neutropenia, otros clostridios (C. septicum)? tiflitis; on pseudo-membranas. Prominentes leucocitosis. Dg: 50-80% PMN, endoscopia es dx de eleccin rpida biopsia es cuestionable. TC en neutropenicos: ciego engrosado. Tto: Oral Flagyl, alternativas para el fracaso, VO Vanco (costoso, mal

sabor) o bacitracina, tambin considerar la nitazoxanida y rifaxamin

*Verdadero o falso: La diarrea est siempre presente en una persona con la amibiasis heptica.*

Falso. El absceso heptico puede ocurrir sin la diarrea o la presencia de la ameba en las heces. Fiebre, dolor en CSD (cuadrante superior derecho), y prdida de peso es ms comn. El organismo entra en la vena porta desde el intestino para infectar el hgado. El diagnstico es generalmente por los ttulos de Entamoeba histolytica, ocasionalmente por el aspirado hepatico: pasta de anchoas pus.

*Verdadero o falso: El absceso amebiano puede tratarse con un medicamento*

Falso. Portador de quiste asintomtico puede ser tratado con un agente luminal, como diloxanida, paromomicina, o iodoquinol. La colitis amebiana y el absceso hepatico deben ser tratados con un agente de trofozoito (metronidazol) y un agente luminal x 10 das.

**

*Cules son las caractersticas de la colitis amebiana?***

Sangre (o hemo +) las heces y fiebre en un viajero del mundo en desarrollo. Frasco en forma de lceras en el colon (biopsia del borde de trofozotos). Huevos en heces y examen de parsitos x 3 se debe hacer para buscar quistes y trophs que contengan hemates ingeridos.**

*Cules son las tres causas de adenitis mesentrica?*

Enfermedad de Crohn Yersinia pseudotuberculosis (pseudo-apendicitis, tambin las lceras de la mucosa en el leon terminal) TB

*Qu enfermedad extraintestinal puede seguir a una infeccin por Yersinia enterocoltica?*

Artritis Reactiva, y el sndrome de Reiter s

*Qu enfermedad extraintestinal puede seguir la infeccin por Campylobacter jejuni?*

Sndrome de Guillain-Barr

*Qu tratamiento adyuvante no debe ser usado con Vanco VO en el tratamiento de colitis por C. difficile?*

Colestiramina VO o colestipol. Se unen a la vancomicina VO. De lo contrario estas resinas son un complemento til al metronidazol VO en la unin de la toxina que causa la diarrea y pseudomembranas. Los probiticos como las preparaciones de Lactobacillus pueden ayudar a repoblar el intestino con flora ms normal que fuera de la competencia del C. difficile. Estos usualmente no son especies de Lactobacillus humano y el intestino eventualmente se repoblara con flora una vez que se vuelva a llenar el tiempo con la flora normal una vez que la influencia del antibitico es suspendida.

*Cualquier problema de diarrea puede ser tratados con anti-espasmdicos tal como la loperamida (Imodium ).** **Verdadero o Falso?*

Cuidado! Si la causa de la diarrea es probable que sea infecciosa o relacionada con toxinas, EVITAR agentes antiespasmdicos. Si las heces acuosas son graves, tratar con rehidratacin oral y agua / aglutinantes de la toxina, tales como colestipol o psyllium, incluso, en su lugar. Recuerde que una funcin de la diarrea es liberar el intestino de los agentes agresores o toxinas. Si ud retarda el proceso bajo el marco del Clostridium difficile o un patgeno invasor, tales como Shigella,

puede precipitar el desastre de megacolon txico (o colitis txica sin megacolon), el cual puede requerir reseccin urgente de todo el colon. Otros medicamentos que pueden precipitar la colitis txica incluyen anticolinrgicos, opiceos y antidepresivos.

**Qu* agentes infecciosos pueden estar asociados con la colitis txica?*

Los patgenos invasivos y / o inflamatorios. Los signos pueden incluir leucocitos fecales +, pequeas heces sanguinolentas (disentera), fiebre, leucocitosis: Clostridium difficile Salmonella, Shigella, Yersinia, Campylobacter Entamoeba histolytica Citomegalovirus

**

*Cules son el 1) cuadro clnico de E. coli H7: 0157, 2) fuentes de infeccin, 3) complicaciones asociadas, 4) medios de cultivo, y 5) el tratamiento?*

Diarrea acuosa sin fiebre, y luego con sangre Hamburguesa de carne media cruda (rosa), sidra de manzana sin pasteurizar Sndrome urmico hemoltico (trada de insuficiencia renal aguda, anemia hemoltica microangioptica y trombocitopenia, nios de 5-10 anos y personas de edad)

El sorbitol-MacConkey (SMAC) de agar (que es sorbitol-negativas / requiere sorbitol para crecer) De apoyo (antibiticos puede aumentar la produccin de toxinas, aumento del riesgo de sndrome urmico hemoltico)

**

*Cules son las fuentes habituales de un absceso del psoas?** *

*Local:*

**Tripa: diverticulitis, enfermedad de Crohn (polimicrobiana)

GU: El absceso perirrenal (Staphylococcus aureus., GNRs) **

*Hematgena:*

** Sangre: bacteriemia desde otro enfoque (S. aureus)

Hueso: tuberculosis vertebral (enfermedad de Pott) fue una la causa ms frecuente**

* *

*GINECOLOGIA Y UROLOGIA*

*Qu infecciones del tracto urinario requieren una mayor evaluacin y un tratamiento prolongado?*

Infecciones urinarias complicadas: ITU en un varn es una infeccin urinaria complicada por defecto. IU no es comn en los hombres debido a una gran distancia entre el meato uretral y la vejiga.Si un hombre se presenta con bacteriuria y piuria, independientemente de la falta de sntomas, dos semanas de antibiticos se debe dar y la evaluacin GU est garantizado. Las causas de infeccin urinaria en los hombres incluyendo: Hipertrofia prosttica benigna Nefrolitiasis La prostatitis crnica Instrumentacin / cateterismo Anomalas congnitas del tracto urinario

*Infecciones urinarias recurrentes.** **Puede ocurrir en hombres o mujeres:***

Inadecuada terapia de antibiticos / microorganismos resistentes Anomalas congnitas del tracto urinario Piedras (urea-que parte de Proteus / Klebsiella spp pueden producir

grandes clculos en asta de ciervo) Uremia / azoemia / necrosis papilar (mala funcin renal no permite que los antibiticos adecuados en el rin de orina) Perivesical absceso o una enfermedad del colon (CA, dz de Crohn) con fstulas a la vejiga HBP / prostatitis**

*Qu hacer con piuria asintomtica y + cultivo de orina en una mujer?*

Si ella no est embarazada, nada. La bacteriuria asintomtica en el embarazo es una de las causas del trabajo pielonefritis, aborto involuntario, y prematuros y garantiza un tratamiento inmediato agresivo.

*Qu hacer con bacteriuria asintomtica (con piuria) o infecciones urinarias recurrentes en un hombre?*

Piense obstrucciones pueden corregir a la prstata agrandada flujo urinario-, las piedras, reflujo ureteral y solucionar los posibles problemas. instrumentacin recientes puede provocar esto tambin y puede causar urosepsis. Las infecciones sintomticas dar lugar a pielonefritis y urosepsis en la obstruccin al flujo de antibiticos y garantiza (recuerde: corregir el flujo).

*Qu hacer con bacteriuria asintomtica en un paciente con una sonda vesical de 2 semanas?*

Nada. Cambiar el catter slo si hay obstruccin del flujo o si hay una infeccin sintomtica (especialmente la levadura). La vejiga o riegos catter bolsa con agua oxigenada, antibiticos tpicos hasta el meato uretral, o cambios de rutina del catter no se asocian con infecciones reducido, y se rompe en el sistema de catter cerrada se sabe que producen la infeccin.

*4 causas infecciosas comunes de infeccin urinaria Cul es asociados** **con clculos renales?***

E. coli Enterococos (estreptococo del grupo D) Estafilococos saprophyticus Proteus mirabilis (indol -) Asociacin con estruvita litiasis renal (clculos en asta de ciervo)

*La vaginosis bacteriana (VB) es una infeccin. Verdadero o falso?*

** No. Es un desequilibrio de la flora vaginal del predominio habitual de las especies de Lactobacillus a una mezcla polimicrobiana de anaerobios y bacilos gram negativos, incluyendo Gardnerella vaginalis, Bacteroides, estreptococos viridans, Fusobacterium, Veillonella, Eubacterium, y Mobiluncus especies.Mycoplasma hominis, Ureaplasma urealyticum y vaginas Atopobium tambin estn asociados con la VB. Aunque los mecanismos no estn claros, los organismos se hacen sentir a crecer de forma sinrgica para ms numerosos que los lactobacilos.G. vaginalis tambin produce un biofilm en la vagina que puede facilitar la colonizacin por otros organismos y se resisten a metronidazol. Un pH bajo tiende a retardar el crecimiento de muchas bacterias, por lo que especulan que una vez que se eleva el pH, el medio ambiente progresivamente se desplaza a uno que es ms amigable para los no lactobacilos y promueve un ciclo de sobrecrecimiento bacteriano.

pH vaginal normal se mantiene en el rango de 3.5 a 4.5 por el perxido y cido lctico-producting lactobacilos. Cuando crecen en exceso de otras especies, el pH sube por encima de 5. BV puede ocurrir en individuos virginales, sin embargo, es altamente asociada con el nmero de parejas sexuales de por vida, una nueva pareja sexual, y la colonizacin uretral masculino con Gardnerella. Mi hiptesis es que las parejas sexuales pueden simplemente introducir una mayor variedad de potencialmente colonizar la flora en la vagina, el perineo. Los condones pueden reducir la incidencia.Parece factible que los cambios hormonales que ocurren durante toda la vida tambin puede alterar la qumica vaginal.

[Verstraelen H. ltimo de lo ltimo: la microflora vaginal y la vaginosis bacteriana. K Verh Acad Belg Geneeskd. 2008; 70 (3) :147-74. PMID: 18669158]

*Aparte de la molestia, cul es la importancia de la VB?***

BV en el embarazo se asocia con una mayor tasa de aborto involuntario, temprano (prematuro) de entrega, y la endometritis post-partum/post-abortion, por lo que es importante para las mujeres embarazadas a someterse a pruebas y tratamiento de la vaginosis bacteriana, aunque asintomticos.

*Cules son los sntomas y hallazgos clnicos de la VB?*

Delgado, homogneo, blanco con gris de descarga que cubre la pared vaginal (biofilm?) Y es molesto para el paciente desagradable olor a pescado, especialmente despus del coito pH vaginal de lquido ms de 4.5 (de alta sensibilidad) uso de una tira de pH Positivo tufillo de prueba (muy especfico) o Aadir una gota de solucin de KOH al 10% a las secreciones vaginales de una diapositiva. Sniff, un olor a pescado indica liberacin de aminas

relacionados con los productos anaerbicos. Patognomnico clulas clave en una preparacin en fresco de secreciones vaginales. Mezclar las secreciones con una gota de solucin salina en un portaobjetos, aadir un cubreobjetos, y ver bajo el microscopio. Las clulas epiteliales cubiertas de bacterias adheridas (efecto biofilm?) son la clave. puede ser excluido por conveniencia, si el pH y olor se +. tincin de Gram del frotis vaginal, clulas epiteliales, el ser salpicado de bacilos Gram negativos. Por lo general no se hace debido a las restricciones reglamentarias y porque tiene que ser enviada al laboratorio.

*El tratamiento de la vaginosis bacteriana:*

*Segn los libros de texto y exmenes de la junta*

Metronidazol 500 mg PO dos veces al da x 7 das El metronidazol en gel intravaginal un aplicador QHS x 5 das o Incremento de la resistencia puede ser un problema, especialmente. Atopobium especies Clindamicina 300 mg PO dos veces al da x 7 das Clindamicina 2% crema vaginal de 5 g por va intravaginal QHS x 5 das Clindamicina vulos 100 mg por va intravaginal QHS x 3 das *Segn Gompf** * 3% de perxido de 30 cc por va vaginal a travs de pera de goma al da x 5 das

barato, de venta libre No vulnerables a la resistencia mnimo de efectos secundarios, sin necesidad de parar la actividad sexual (el exceso de lquido se ejecutar de nuevo en la cmoda, ya que puede ayudar a alcanzar plenamente todas las reas, si el paciente se encuentra en piso en un par de minutos despus de infundir el lquido.) Obras-por qu no es el conocimiento comn o bien estudiado? Debido a que est enterrado en PubMed, y no hay patrocinador farmacutico (sin fines de lucro en perxido!). Ver Cardone A, et al. La utilizacin de perxido de hidrgeno en el tratamiento de la vaginosis bacteriana recurrente. Minerva Ginecol. 12 2003; 55 (6) :483-92. Examen. PMID: 14676737. Llanura Stoneyfield o yogur Dannon marca (es fiable cida y constantemente se activa culturas-No tengo ninguna relacin financiera a revelar, pero soy un experto fabricante de yogur!) 2-3 cucharadas (diluido con agua destilada o solucin salina si la consistencia es muy espesa) que se aplica por va intravaginal QHS x 6 das baratos, de venta libre, sin efectos adversos, sin resistencia, sin necesidad de parar la actividad sexual (pareja puede saber sobre l, sin embargo, para evitar la incomodidad). Repoblacin o por los lactobacilos puede ser facilitado por el perxido de pre-tratamiento mediante la restauracin de un entorno favorable para ellos. Usted ser una estrella para la mujer que viene a usted con su ensimo episodio BV hartos de antibiticos.

*CABEZA Y CUELLO*

*Cul examen del LCR es poco apreciado (y subutilizados), pero tan til como la tincin de Gram en determinar la presencia de meningitis bacteriana vs asptica?*

* *El nivel de lquido cefalorraqudeo de cido lctico. El cido lctico LCR <3 mmol = meningitis asptica, 10 mmol la meningitis bacteriana =. 3.10 mmol por lo general la meningitis bacteriana.es parcialmente tratada. **

*La meningitis bacteriana se inicia en los siguientes sitios con los agentes patgenos siguientes:*

Listeria infeccin gastrointestinal asintomtica o leve Neumococo neumona, otitis media / sinusitis Hemophilus-otitis media Estafilococo dorado endocarditis Meningococo faringitis

*Adems de cubrir el neumococo y el meningococo, que organismos tu cubres en pacientes mayores de 50 o embarazadas?** **Qu aadirias a la pauta inicial?*

Listeria (ampicilina o Septra)

*Si el laboratorio te informa acerca de diptheroids en la muestra de lquido cefalorraqudeo, qu pensarias?*

Listeria, Listeria, y Listeria!

Es un Bacilo Gram +, como Bacillus y corinebacterias, y puede ser confundido con los contaminantes de piel.

*Qu organismo es importante en la causa de la meningitis en pacientes con neurociruga, fstulas de lquido cefalorraqudeo (rinorrea clara) o fractura de base de crneo?*

Neumococo. Tambin S. aureus, Pseudomonas / GNRs

*Encefalitis del Nilo Occidental a menudo se presenta con una parlisis ascendente.** **Cmo podrias diferenciar la Encephilitis del Nilo Occidental del sndrome de Guillain-Barr?*

*Virus del Nilo Occidental

Confusin

Pleocitosis en el LCR

Personas mayores Fiebre

*Sind Guillian-Barre*

Sensorio claro

LCR normal

*Causas bacterianas clsicas de la faringitis y droga de eleccion para cada uno.*

Estreptococos del grupo A (PNC / claritromicina x 10d-hay una creciente resistencia a los macrlidos) Arcanobacterium haemolyticus (P N C/ macrlidos) Neumona por Mycoplasma (Doxy / macrlidos / quinolonas) Corynebacterium diphtheriae (eritromicina + antitoxina de los Centros de Control de Enfermedades)

*Qu causa la epiglotitis?*

Nios < 5: Hemophilus (desapareciendo debido a la vacuna contra Hib(Hemophillus influenza tipo B)), incrementndose el estreptococos del grupo A Nios> 5: Estreptococos del grupo A Adultos: Casi siempre estreptococos del grupo A, Haemophilus (en los adultos mayores)

*Qu organismo causa la uvulitis aislado (inflamacin de la vula)?*

Estreptococos del grupo A de color rojo + / Exudado**

*Qu otra cosa es en el diagnstico diferencial de la uvulitis?*

Trauma (intubacin, aspiracin agresiva durante los procedimientos) Angioedema o reacciones alrgicas? vula plido

*Cules son las secuelas inmunolgicas de las infecciones por estreptococos del grupo A y cmo podran prevenirse?*

*Fiebre reumtica aguda, enfermedad reumtica del corazn*

**- slo faringitis estreptococica

-PNC es preventivo, por lo tanto siempre tratar la faringitis estreptoccica para prevenir la fiebre reumtica!

*Glomerulonefritis Aguda post-infecciosa * -Faringitis estreptococcica e imptigo

-PNC no lo previene

*Qu complicaciones / sntomas estan asociados con infeccin en cuales senos paranasales?*

Frontal absceso subdural/ epidural -tumor hinchado de Pott Maxilar entumecimiento facial leve (segunda rama del nervio craneal V), dolor de dientes Etmoidal lagrimeo, trombosis del seno cavernoso Esfenoidal retroorbital / occupital cefalea, celulitis orbitaria

*Cmo diferenciar la celulitis orbitaria de la trombosis del seno

cavernoso?*

Trombosis del seno cavernoso: edema de papila, afectacin ocular bilateral, Puncion Lumbar anormal, parlisis del V par craneal

*Cules son las complicaciones de la otitis media crnica?*

Colesteatoma Mastoiditis / ostetis mastoides Parlisis del V nervio facial (requiere urgente miringotoma) Meningitis Absceso epidural Empiema subdural Un absceso de Bezold es un absceso en el cuello desde la mastoiditis.

*Lista de las 10 principales causas del derrame crnico otico.*

Mastoiditis Cuerpo extrao Colesteatoma Cncer Histiocitosis X Sfilis (tratada como neurosfilis) Tuberculosis Munchhaussen de

Pseudo-Munchhaussen de Otitis externa maligna Derrame de LCR

*Cules son los resultados y asociaciones clsicas con otitis externa maligna?*

Otalgia, otorrea, y tejido de granulacin en el conducto auditivo externo en pacientes diabticos. parlisis del nervio facial. Las complicaciones incluyen la osteomielitis del hueso temporal (ver TC del hueso temporal), requiere desbridamiento. Pseudomona es el Organismo.

*3 causas de la meningitis bacteriana y Droga de eleccion:*

H. gripe. Ceftriaxona N. meningitidis PNC Listeria monocytogenes Ampicilina / Gentamicina Neumococo Vancomicina + Ceftriaxona

*Diferenciar entre la tuberculosis, herpes, cryptococcosis y

toxoplasmosis del sistema nervioso central*:

*Toxo* mltiples lesiones en el parnquima cerebral, signos neurofocales. *Cryptococcus *- indolente, meningitis purulenta, aumento de la presin intracraneal, SIDA. *Herpes *- asoc. con LCR hermorrhagico y hipoglucorraquia, (disminucin de la glucosa en el LCR), aumento del lbulo temporal en la RM. *TB *- linfocitosis en el LCR con hipoglucorraquia, aumento menngea basilar en la RM, el aumento de la presin intracraneal (necesidad de los esteroides)

*Cules son los factores de riesgo para la meningitis y cual patgeno?*

Fuga post-traumatica de LCR / lfractura de la placa cribiforme-neumococo Crneo/fracturas del seno, incluso a distancia- el neumococo Queso Sin pasteurizar / quesos blandos / embutidos- listeria Hamsters / roedores virus lymphochoriomeningitis - Lagos de agua dulce caliente Naegleria fowleri Lentes de contacto / lagos de agua dulce calientes / VIH/Acanthameba **

*Cmo sabe usted si el fluido nasal es LCR o no en el lado de la cama,

si el LCR goteado es sospechoso?*

Hacer una prueba de verificacion en l. Si es + para la glucosa, es LCR.

*Que se conoce como el Cinturn de la Meningitis?*

La regin ecuatorial de frica es tan conocida. El meningococo se transmite fcilmente en vientos clidos y secos. Los viajeros a la zona deben recibir la vacuna contra el meningococo.

**

*Cul es el diagnstico diferencial de la linfadenitis necrotizante?** *

Linfoma de Hodgkin y no-Hodgkin Tuberculosis (escrfula) / micobacterias atpicas (historia de exposicin a la tuberculosis, jardinera, etc) La toxoplasmosis (la historia de exposicin al gato) Sarcoidosis**

* *

*CARDIAC**O*

*Lista de las 4 causas mas comunes de la pericarditis infecciosa:*

* *Viral enterovirus, por lo general Coxsackie A y B, influenza A y B Bacteriana meningococo, estafilococo dorado, gripe por Hemophilus en los nios Tuberculosis pericarditis constrictiva Hongos Histoplasmosis, Aspergillus

*Nombre los factores predisponentes para la endocarditis infecciosa:*

Endocarditis previa Vlvula daada nativos *- congnita o reumtica Prtesis vlvulares ESHI (Estenosis Subaortica Hipertrofica Idiopatica) DSV (Defecto Septal Ventricular) * S. aureus (y S. lugdunensis) puede infectar una valvula sana.

*Nombre dos tipos de endocarditis infecciosa:*

Liebmann-Sacks asociados con LES (sin infeccin, sin embolias, no ABT) Marantic: asociados con malignidad**

*Lista 7 estigmas perifricos de endocarditis:*

Petequias (membranas mucosas) Puntos de Roth (hemorragias retinianas) Bastones Hemorragias en astilla (matriz de las uas) Lesiones de Janeway (lesiones planas, indoloras en las palmas de las manos/ plantas de los pies) Nodulos de Oslers (palpable, dolorosa nodos en la punta de los dedos) Esplenomegalia**

*NOTA*: En la era de los antibiticos, esto ocurre en aproximadamente en un 15%. Ausencia no I / O SBE.

*Para cada configuracin especial en el cual ocurre la endocarditis bacteriana, la lista de patgenos asociados:*

Los drogadictos Staphylococcus aureus / Candida parapsilosis Prtesis Vlvular Staph. epidermidis, hongos Cultivos negativos La fiebre Q (Coxiella burnetii), grupo HACEK (gram-negative bacilli:/Haemophilus/ species /(Haemophilus parainfluenzae, Haemophilus aphrophilus, Haemophilus paraphrophilus), Actinobacillus actinomycetemcomitans, Cardiobacterium hominis, Eikenella corrodens,/ and /Kingella/ species) Cncer de colon Estreptococcus bovis

*Cules son los criterios de Duke modificados para el diagnstico de endocarditis infecciosa?*

2 criterios mayores 1 criterio mayor + 3 criterios menores, o 5 criterios menores

*CRITERIOS MAYORES* 1. + BC SIN FUENTE: 2 barras de 12 horas de aparicion o 3 tallos de a 1 hora de aparicion (bacteriemia continua): Estreptococo viridans, Estreptococcus bovis Enterococcus HACEK bacterias Estafilococo aureus

2. + Eco: vegetacin absceso perivalvular nueva dehiscencia de la prtesis valvular nueva regurgitacion valvular

3. 1 + w AC / Coxiella burnetti o + ttulo> 1:800

*CRITERIOS MENORES* 1. Enfermedad reumtica del corazn Hx / condicin predisponente / Abuso de Drogas Intravenosa (ADIV) 2. T> 38 C / 100.4 F 3. Fenmenos vasculares / embolicos (Embolia, hemorragia ACV, hemorragias de uas, hemorragias conjuntivales) 4. fenmenos inmunolgicos (Ndulos de Osler, manchas de Roth, glomerulonefritis, factor reumatoide +) 5. + BC x 2 W / organismos distintos de los anteriores 6. Endocarditis sugestiva de Echo (pero no los criterios mayores)

*Lista 7 complicaciones de la endocarditis:*

ICC Defectos de conduccin (absceso del anillo valvular) Aneurismas micticos (no por hongos , en forma de hongo) IM (secundaria a embolia coronaria) Glomerulonefritis focal y difusa Obstruccin (valvular) Accidente cerebrovascular emblico Absceso Emblicos / infx: cerebro / bazo (lado izquierdo), pulmn (lado derecho)

*Cules son los patgenos usuales en la endocarditis bacteriana?*

Estreptoccica = 70% (estreptococo viridans>> enterococos del Grupo D) Estafilococo aureus = 20% Miscelaneos (gripe por Haemophilus influenza, Pseudomonas, Bacilos Gram Negativos, Pneumococos)

*Cules son dos signos de alerta de la presencia de una Endocarditis Bacteriana *

**Fiebre Murmullo (85%)

*Tratamiento de la EB:*

De acuerdo a la sensibilidad del aislamiento del hemocultivo (HC). Antes de los resultados del hemocultivo, el tratamiento con nafcilina / oxacilina + Gentamicina.**

*Lo caracterstico de la endocarditis por hongos?*

Vegetaciones grandes Gran embolia arterial (por ejemplo, los pies sin pulsos y fros en el UDIV)

Endocarditis por hongos es una indicacin absoluta para el reemplazo valvular. Tratar con anfotericina B intravenosa IV y la ciruga lo antes posible.

*Quin est en riesgo de endocarditis micotica?*

Uso de Drogas IntraVenosa (20% de los UDVP endocarditis por hongos) Inmunosupresin Post Ciruga cardaca

*Qu hongos estn asociados con endocarditis y en cuales individuos?*

UDIV-Candida parapsilosis Inmunosupresin Aspergillus Post-ciruga cardaca spp Candida.

*Qu condiciones cardacas NO requieren profilaxis de la endocarditis?* Comunicacin interauricular / defecto septal ventricular reparada, persistencia del conducto arterioso despus de los 6 meses, prolapso de la vlvula mitral sin regurgitacin

*PULMON*

*Bactarias causantes de la neumona atpica y drogas de eleccin para cada una:*

Mycoplasma pneumoniae (Macrolidos/Quinolonas/Doxiciclina)

Clamidia pneumoniae (Macrolidos/Doxiciclina/Quinolonas)

Legionela pneumophila (Macrolidos/Quinolonas)

Clamidia psittaci (Doxiciclina)

*Principales causas virales de neumona, y Tratamiento para cada uno.*

Influenza A (Amantadina / inhibidor de la neuraminidasa) Virus sincicial respiratorio (VSR) (ribavirina) Adenovirus (Ninguno) Influenza B (inhibidor de la neuraminidasa)

*Nombre de ocho organismos que causan la neumona.*

Neumococo Haemophylus influenza Klebsiella Mycoplasma Legionella Clamidia Pneumocistis carinii o pneumonia Virus

*Cules son las caractersticas de la neumona por Klebsiella, cual es el tratamiento, y quienes estn en riesgo?*

Esputo como Jalea de grosella (hemorragia / necrosis, gangrena pulmonar rara vez puede ocurrir) Fisura horizontal abultada Infiltrado lobular Doble cobertura contra Gram negativos: penicilinas antipseudomonas (imipenem si es grave / UCI) + quinolona / aminoglucsido (recuerde: Klebsiella puede ser Beta Lactamasa de Espectro Extendido / rpidamente se vuelve resistente) Alcohlicos Diabticos EPOC

*Describir la neumona por aspiracin, microbios y Drogas de eleccin.*

* *Pacientes con convulsiones y alcohlicos que vmitos aspiren material omitado y desarrollen un absceso de cavida en el lbulo inferior (necrosis cida centro-lobulillar), por lo general no de inmediato. (Fiebre, infiltrados blancos / aspiracin aguda se deben a la neumonitis qumica.) Anaerobios son responsables de la infeccin. Tto: PNC / Clindamicina

*Nombre tres organismos que causan la neumona por microaspiracin, sntomas caractersticos, y Drogas de Eleccion.*

Neumococo: comienzo brusco, dolor pleurtico, escalofros, fiebre y temblores; esputo de color oxido; paciente normal neumona lobar Tto: ceftriaxona + azitromicina / claritromicina o quinolonas Haemophylus influenza gripe: principalmente asociado con bronquitis en pacientes con enfermedad pulmonar crnica, EPOC Tto: Ceftriaxona Klebsiella: alcohlicos, drogadictos, DM, EPOC, las personas mayores en hogares de ancianos; esputo amarillo. Tto: 3ra o 4ta generacin de cefalosporinas- imipenem, si esta crticamente enfermo.

*Qu enfermedades se asocian** **con asbestosis y silicosis,?*

Asbestosis cncer mesotelioma de clulas escamosas Silicosis Tuberculosi

* *

*OBSTETRICIA Y NEONATAL***

*Causas de fiebre que no deben ser subestimadas en el perodo post-parto con su tiempo de aparicin.*

Dentro de las 24 horas: sepsis Perpueral (fiebre puerperal) Estreptococo Grp A 24-48 horas: Endometritis asociada con flujo vaginal maloliente 48-72 horas: tromboflebitis plvica asociada con embolia pulmonar sptica.

*C**ausas comunes de la fiebre puerperal en las primeras 24 horas:*

Congestin de las mamas Aspiracin (despus de la anestesia general / Cesarea emergentes) / atelectasia Infeccion del Tracto Urinario

*Lista de los tres perodos que deben considerarse en las infecciones fetales y neonatales y los **virus asociados:*

Congnita CMV, rubola, VHS, VVZ (Virus de la Varicela Zoste)r, virus de la Hepatitis B CMV -Natal, VHS, hepatitis B, enterovirus Despus del parto CMV, hepatitis B, herpes, VSR, Enterovirus

**

*Enumere tres causas de conjuntivitis neonatal y su tiempo de aparicin:*

GC 3-5 das (muy purulenta) Qumica 0-3 das La clamidia 5-14 das

*Qu es el sndrome TORCH?*

* TO*xoplasmosis *R*ubola *C*itomegalovirus *H*erpes simple tipo 2 (y 1)

infecciones congnitas clnicamente similares causadas por Toxoplasma gondii, virus de la rubola, citomegalovirus y el virus del herpes simple, tipo 1 y 2 que se evidencian en el recin nacido por manifestaciones cutneas: petequias, prpura, ictericia, y la eritropoyesis drmica (erupcin muffins de arndanos). El riesgo es mayor si la infeccin materna fue adquirida en el 1er trimestre.

*Nombre los principales antibiticos proscritos en el embarazo:*

*QUITS PREGnanC*

QU quinolonas I interfern T tetraciclinas S sulfas

P PZA R ribavirina E Estolato de eritromicina G Griseofulvina n a n C Cloranfenicol

*Qu hacer si una mujer embarazada tiene antecedentes de herpes genital?*

Cesrea si hay lesiones activas en el momento del parto. Debido a que las madres pueden arrojar VHS sin lesiones activas, hacer vigilancia del recin nacido por va vaginal para lceras orales y del cuero cabelludo, puede ocurrir cientos de partos vaginales con VHS y / o vigilar cientos de bocas de recin nacidos al nacer y durante las primeras 48 horas? si es positivo, dar al neonato 21 das de aciclovir.

*Cul es el riesgo para los recin nacidos que nacen por va vaginal de una madre con herpes genital?*

Herpes neonatal diseminado, lo cual puede ser fatal.

*Qu hacer si una madre en trabajo de parto con 5 das de la varicela en desarrollo, o se desarrolla la varicela hasta 2 das despus del parto?*

Dar al beb IgG contra la varicela para prevenir la varicela neonatal diseminada. Las mujeres en edad frtil (especialmente los trabajadores de salud) deben recibir la vacuna contra la varicela antes del embarazo si hay antecedentes de varicela.

*Cul es el riesgo ms grave de la varicela en adultos, la ms alta para las mujeres en el 3er trimestre del embarazo?*

Neumona por varicela. Dar IgG contra la varicela si la madre no inmune se expone durante el embarazo. Darle la vacuna contra la varicela despus del parto.

*Una mujer en su 1er trimestre del embarazo se expone a la rubola en su dormitorio de la universidad.** **Ella no recuerda su historia de inmunizacin.** **Qu hacer?*

Esta es la razn por la cual la vacunacin previa a la concepcin es tan importante en mujeres en edad frtil (especialmente en el ingreso a la universidad y en las areas de la salud) y por qu los exmenes fsicos del pre-universitario debe cubrir esto; los dormitorios son ambientes comunes de exponerse a enfermedades prevenibles por vacunacin u otros, por eso estas pueden ser un ambiente de mezcla internacional Compruebe la rubola ttulos; si es negativa, ella tiene 2 opciones: 1. Generalmente, se recomienda / estndar de atencin: Si ella ha sido infectada con rubola, el riesgo de graves defectos congnitos es de 30-50%.Si la madre va a querer un aborto en lugar de correr el riesgo de defectos congnitos, NO debe administrarse IG. Repita los ttulos en tres semanas, el aborto teraputico se ofrece si ella presenta Seroconversion + para anticuerpos.

2. Si la madre no considera el aborto si ella est infectada, la IG pueden ser ofrecidas. La IG generalmente no se administra, ya que no previene muy bien la infeccin de la rubola fetal, y la enmascara si la madre se ha seroconvertido. Podria proporcionar (poco fiables) proteccin al feto.

* *

*PIEL Y TEJIDOS BLANDOS*

*/LESIONES DE PIEL:/*

*Qu tipos de lesiones general de piel existen?*

Tumores Erupciones

*Cules son los tipos de tumores?*

Cutnea: nevus, fibromas, plipos, cncer de clulas basales. Epidrmicas: verrugas, queratosis seborreica, queratosis actnica, cncer de clulas escamosas, queratoacantoma Papuloescamosas (circunscrita): Psoriasis (escamas gruesas), liquen

plano (sin elevaciones). Polymorfica: ppulas crnicas agudas / liquenificacin, elevaciones.

*Describir la dermatitis atpica y su distribucin.***

La picazn que brota en lugar de la erupcin que pica, es decir, pequea lesin papular que resulta en un eritema, exudando, y la evolucion secundaria al rascado. Situado en las zonas de flexin: cuello, pliegues antecubital, popltea, prpados, muecas, detrs de las orejas.

**

*Describir la dermatitis seborreica y su distribucin:*

Escala de parches, los mrgenes indistintos, modo eritema, aceitosa, a menudo amarillo. Localizado en cuero cabelludo, retroauriculares, cejas, prpados, superficies extensoras de los pliegues nasolabiales- y circunscrito en las extremidades distales.

*Describir la dermatitis por contacto y su distribucin.*

Puede ser alrgica o irritante. La piel es tocada por la sustancia. Miles de causas y morfologa posibles. Cara y manos son los sitios ms comunes.

*Describir la dermatitis por estasis y su distribucin.*

rea de eritema ciantico que es pruriginosa, dolorosa, y se encuentra sobre una vena dilatada que con el tiempo resulta en una ulceracin. El sitio inicial ms comn es el rea por encima de malolo medial.

**

*Describir la dermatitis numular y su distribucin.*

Lesiones anulares o circulares que aparecen rapidamente, no cambian de tamao, exudado y picor, ademas la costra y elevacion. Afecta a hombres mayores y mujeres ms jvenes y es exacerbada por el agua caliente. A menudo localizadas en superficies extensoras de las extremidades, en la espalda, glteos y piernas.

*Nombre 6 tipos de infecciones dermicas, sus causas bacterianas, y los

tejidos afectados.*

El imptigo contagioso estreptococos del grupo A epidermis (Asociado con glomerulonefritis) Erisipela estreptococos del grupo A dermis (bordes definidos; edema de piel de naranja) Celulitis estreptococo pyogenes / estafilococos tejido subcutaneo (Bordes difusos y edematosos) Fascitis flora mixta fascia Miositis clostridios muscular Imptigo bulloso Staphylococcus aureus

*Describir infecciones de heridas causadas por estafilococos, estreptococos Grp A y Pseudomonas.** ***

*estafilococo dorado* coagulacin de plasma y necrosis de los tejidos blandos, abscesos bien localizados, sobre todo en cara, cuello, ingle, heridas postoperatorias.

*S**AMR adquirido en la comunidad (nuevos cultivos de SAMR que surgen en una persona que no ha estado institucionalizado o sometido a una ciruga en los ltimos 12 meses, sin dispositivo o sonda vesical permanente que rompa la barrera de la piel)* es ahora prevalente en algunas comunidades en el orden del 60% el SA entra y rpidamente causa la muerte. Caracterizado por una infeccin del tejido blando severa rpidamente

progresiva, incluyendo piomiositis y fascitis necrotizante.

*Factor de virulencia *= *citotoxina leucocidina Panton-Valentine. *Una idea comn que se presenta es el informe del paciente de una *Picadura de araa,* incluso cuando no fue visto un arcnido, presumiblemente porque la causa de la aparicin sbita de dolor agudo e inflamacin que puede sugerir que esto ocurra- pensar en SAMR-AC*.* Ver fascitis necrotizante mas adelante.

*Tto*: TMP / SMX o doxiciclina / minociclina VO si es leve (organismo puede demostrar la resistencia inducible a clindamicina, han realizado pruebas de laboratorio antes de confiar en este agente), si enferma sistmicamente tigeciclina / vancomicina IV, daptomicina IV / linezolid IV o VO son opciones mucho ms costosas.

*Strep A* protena M = factor de virulencia; inicio temprano; invasin rpida, celulitis de rpida evolucin; bacteriemia que amenaza la vida, bacteria carnvora Tto: PCN-G + (GAS sigue siendo altamente susceptibles a la PCN-G) / Clindamicina (clinda inhibe la protena M detiene la sntesis de protenas); considerar una carbapenem en casos donde la fascitis necrosante polimicrobiana es probable, tal como la gangrena de Fourniere (infeccin escrotal en diabticos) o despus de la ciruga intraabdominal.

*Pseudomonas* generalmente w / infecciones mixtas; pacientes quemados, pacientes gravemente enfermos; herida extendida; gran cantidad de tejido necrtico; humedad tpica / dulce / mal olor (como el jugo de uva en cultivo puro), coloracin azul-verdoso; septicemia grave. Tto: quemadura Sulfamylon topica, Silvadene y 0,5% de nitrato plata; tazobactam piperacilina- sistmico + tobra / ciprofloxacino

**

*Describir las dos formas de celulitis crepitantes, as como la gangrena sinrgica.***

**

*La fascitis necrotizante*

*Aguda infeccin* mixta Strep A, Bacteroides, estreptococos anaerobios, Clostridium. SAMR-AC

*Indicios clnicos claves de fascitis necrotizante.*

1. DOLOR (profundo y con frecuencia fuera de proporcin a los resultados) + FIEBRE. 2. Textura leosa induracin de la zona dolorida 3. Anestesia cutnea sobre la zona dolorida (inervacin cutnea es transmitida como la fascia muerta) 4. Bullas violceas

** 50% del tiempo, fascitis necrotizante no presentan w signos clnicos de infeccin sobre el area dolorosa .**

Tto: reseccin amplia y descompresin extensa; PNC IV + clindamicina Hiperbrica O2/IV IG pueden ser complementos tiles.

*Gangrena gaseosa por clostridios:* C. perfringens / C.septicum sepsis

Fiebre, dolor severo, MS Antecedente de trauma, fracturas abiertas, postaborto / postparto (dama rosa-hemlisis), o cancer subyacente de colon Bronce o el aspecto rosado de la piel por la hemlisis masiva

*gangrena sinrgica progresiva de Meleney*: forma progresiva crnica de una infeccin mixta, estreptococos (no hemolticos, microaerfilos) y estafilococo dorado; Comienza alrededor de bordes de la herida / ostomas. Lesin = celulitis rojo palido con centro prpura, progresivamente tornandose gangrenoso, ulceracin, mrgenes dolorosos, grisaseo con prpura que se extienden. Tto: escisin amplia, PNC+ EMYCIN**

*Qu es piomiositis?*

Anteriormente encontrado principalmente en los trpicos, ahora frecuentemente causadas por cepas SAMR-AC + por la toxina Panton Valentine leucocidina (PVL +). Aparicin sbita de fiebre, dolor muscular severo/ infeccin profunda, y abscesos, que no siempre puede ser evidente en la superficie. Tratar con desbridamiento quirrgico agresivo y antibiticos por va intravenosa para cubrir SAMR-AC (Ver Describir infecciones de heridas causadas por estafilococos, estreptococos Grp A y Pseudomonas anteriormente en esta seccin).

**

*La gangrena gaseosa se produce slo con algn tipo de trauma?** **Verdadero o falso.*

Falso. Clostridios (especialmente C. septicum) gangrena puede ocurrir espontneamente, especialmente en pacientes con CA de intestino o neutropenia. Tambin podra recurrir (esporas pueden persistir en el tejido infectado previamente!).**

*Que es muy importante recordar al tratar la piel sensible o infecciones de tejidos blandos con cefalexina (Keflex?)?*

Necesita del cido estomacal para ser absorbido. Muchos pacientes que hoy estn en los reductores de cidos (con receta o sin receta). No usarse en pacientes que estn en bloqueo H2 crnico, inhibidores de la bomba de protones o usan anticidos con frecuencia. Considerar la

posibilidad de clindamicina o doxiciclina en pacientes que son alrgicos a la penicilina, como una alternativa.

**

*Causas de gas de tejidos blandos:*

Clostridium perfringens / septicum Anaerobios

**

*Cules son las causas de linfangitis nodular?*

Esporotricosis (espinas de las rosas, musgo Sphagnum) Nocardia (brasiliensis, esp.) Mycobacterium marinum (granuloma en tanque de peces); tambin M chelonei/ fortuitum Leishmaniasis cutnea (Leishmania brasiliensis; trpicos) Francisella tularensis (tularemia ulceroglandular o glandular)

*Cul es el diagnstico diferencial de linfaadenopatas pre-auricular?** **(Sndrome oculoglandular de Parinaud?)*

TB dacrocystitis

Conjuntivitis adenoviral, fiebre faringoconjuntival Chlamydia trachomatis/ conjuntivitis por gonococos Enfermedad por araazo de gato Tularemia oculoglandular Enfermedad de Chaga (Trypanosoma cruzi) Linfoma Hidrocefalia obstructiva aguda**

**

*Cules son las causas de edema periobital?*

Triquinosis (eosinofilia, elevacin de CPK, bajo rata de sedimentacin eritrocitaria) la enfermedad de Chaga (aguda signo -> Romana) Dermatomiositis (erupcin heliotropo)

*Qu organismos son conocidos por producir el sndrome de shock txico?*

Staphylococcus aureus (endotoxina, TTS-1) Estreptococos del grupo A (exotoxina, exotoxina estreptoccica pirognica A)

*Qu es el sndrome de shock txico y cmo puede diferenciarse el shock txico por estafilococos del de estreptococo?*

Staph TTS se caracteriza por shock sptico y erupcin eritematosa difusa, como una quemadura de sol, y se asocia con: Heridas (postoperatorio especialmente, aunque no se ven infectadas) Tampones vaginal muy absorbentes El taponamiento nasal (pensar, tapn nasal) por la epistaxis (Otorrinos usualmente dan antibioticos profilcticos; la sangre proporciona un medio de cultivo perfecto para el estafilococo dorado, que normalmente coloniza las fosas nasales anteriores) Estreptococo TTS se caracteriza por shock sptico y una infeccin profunda de tejidos blandos, especialmente fascitis necrotizante. Considere la posibilidad de IgIV.

**

*Patgeno que causa la celulitis facial prpura en ancianos.*

Haemophilus influenza (desapareciendo en nios desde la introduccin de la vacunacin contra HI)

**

*Causa de erisipela:*

Erysipelothrix rhusiopathiae (pescado crudo, pollo) PNC

*8 causas de la lcera que no cicatriza:*

Pioderma gangrenoso Cuerpo extrao Vasculitis Autoinmune Micobacterias Gomas sifilticas Carcinoma Enfermedad de Munchaussen (auto mutilacin)

*3 causas infecciosas de la cara roja:*

shock txico: estafilococo aureus (toxina) Escarlatina: estreptococo del grupo A La quinta enfermedad: Parvovirus B-19

*Nombre ocho condiciones dermatolgicas que comiencen por E, asociado a un proceso infeccioso y el agente responsable:***

**

*Eritrasma:* Corynebacterium minutissimum confunde fcilmente con la infeccin por hongos en la piel de color rosa debajo de la lmpara de

Wood. La infeccin se ve en el rea de la ingle. *Tto:* EMYCIN* *

*Eritema infeccioso:* Parvovirus B-19 La quinta enfermedad apariencia de una bofetada en la mejilla en la cara de los nios. Lividices reticularis en la parte inferior del cuerpo.

*Eritema marginado:* fiebre reumtica estreptococo beta-hemoltico del Grp A

*E**ritema nudoso:* Toda enfermedad granulomatosa ubicado en la cara anterior de la pierna- etiologas infecciosas multifactorial -A menudo, en VHS, Mycoplasma, histoplasmosis

*Eritema nudoso leproso (ENL):* lesiones nodulares dolorosas en extremidades en la lepra, reaccin de Arthus a la lepra, tratamiento con talidomida.

*Eritema multiforme:* Reaccin de hipersensibilidad mculas, vesculas, en extremidades distales; lesiones anulares en los labios.

*Eritema migrans:* Enfermedad de Lyme aspecto de ojo de buey Borrelia burgdorferi

**

*Nombre 4 medicamentos asociados con el sndrome de Stevens-Johnson:*

Dilantin

Fenobarbital Sulfonamidas (Septra) Fansidar (Pyrimethmine / sulfadoxina) para la malaria por P. falciparum resistente a la cloroquina*.***

**

*Cules son las causas infecciosas comunes del eritema nudoso?** *

*Mycoplasma VHS*, VEB, Sarampin Histoplasmosis, coccidioidomicosis Enterovirus BCG (Mycobacterium bovis), M. Tb. Pneumococo, Enterobacter Vacuna contra la viruela (viruela de las vacas-vaccinia virus se utiliza en la vacuna)

**

*Cules son las causas no infecciosas del Eritema nodoso.?*

Igual que la de Stevens-Johnson: sobre todo las drogas.

*Lista 5 caractersticas de una herida tetangenas:*

1 cm de profundidad Duracin 6 horas Dimensin puncin de la herida estrellada Sucio contaminados / cuerpo extrao tejido desvitalizado / quemaduras

*Enfermedades que se confunden fcilmente:*

Artritis reumatoide juvenil + enfermedad de Lyme

Fiebre tifoidea (Salmonella typhi) + tifus (Rickettsia prowazekii)

* *

* *

*ENFERMEDADES DE TRANSMISION SEXUAL***

*Lista 4 comunes enfermedades de transmisin sexual, agente causal, y l droga de eleccin (DDE):*

UNG (uretritis no gonoccica) (Chlamydia) -Azithromycin/Doxycycline Sfilis (Treponema pallidum) (chancro duro, sin dolor) PNC

Gonorrea (GC) ceftriazona Chancro blando (Haemophilus ducreyi) (chancro blando, doloroso) azitromicina / Ceftriaxona.

*Cules son las manifestaciones oculares de la sfilis?*

Pupila de Argyll-Robertson: Piensa en una prostituta, pero no tiene capacidad para reaccionar

Visin Gun Barrel (tnel) Pensar casamiento a la fuerza?

*Cules son las causas de un falso + RPR o TLC?*

Muchos (de cualquier proceso que implica un fuerte estmulo inmune humoral), tales como infecciones crnicas, bacteriemia recurrente, exposiciones peridicas a los antgenos (transfusiones de sangre, multiparidad), etc

*Causa infecciosa de un RPR + / TLC neg:*

Leptospirosis Fiebre recurrente

Lepra Fiebre por mordedura de rata (Spirillum menor de edad) TB Neumona Pneumonococcal Neumona por micoplasma Varicela Endocarditis bacTerial subaguda Chancroide Escarlatina Enfermedades por Rickettsias Malaria Tripanosomiasis Vacuna contra la vaccinia (virus vivo) Sarampin Linfogranuloma venreo Hepatitis Mononucleosis infecciosa Principios de la infeccin por VIH

*C**ausas no infecciosas de un RPR + / TLC neg:*

Uso de Drogas IntraVenosas Cualquier trastorno de la enfermedad conjuntival Enfermedad cardiaca reumatoidea LES Transfusiones de sangre (mltiple)

Embarazo (especialmente las mujeres multparas) La vejez Enfermedad crnica del hgado

*/ /*

*Causa infecciosa de un TLC + / RPR neg:*

Enfermedad de Lyme

*En qu casos es un TLC + solo justificacin suficiente para el tratamiento de la sfilis con PNC-G IV?*

En el caso de la sfilis ocular (uvetis, atrofia ptica/ neuropata, coriorretinitis) y otitis sifiltica, los cuales pueden presentarse sin otras manifestaciones de neurosfilis y el progreso a la ceguera / sordera si no se trata. Ambos son a menudo de larga duracin y RPR, normalmente utilizado en los controles, puede ser negativo. Oftalmologa / tratamiento recomendado en la literatura ORL ambos de estos dos sndromes como neurosfilis activa sobre la base de un acuerdo de libre comercio + solo si no se encuentra otra causa: PNC IV durante 14 d. Para otitis, hay algunos datos usando esteroides y una terapia ms prolongada.

* *

**

*NO INFECCIOSAS Y HALLAZGOS DE LABORATORIO*

**

*Lista de causas no infecciosas e infecciosas con alta velocidad de sedimentacin globular (> 100 *):*

Osteomielitis ACG / PMR CVDz MM / HL Tiroiditis subaguda Infarto tisular / necrosis Infarto agudo de miocardio / tromboflebitis / trombosis Insuficiencia renal crnica Cancer Embarazo Reacciones de hipersensibilidad a drogas

* Nota: El sndrome nefrtico y la enfermedad renal en etapa terminal puede estar asociada con VSG> 100 en un 20% de los casos.

*Cules son las causas no infecciosas comunes de una leucocitosis neutroflica?*

-Trauma / Destruccin tisular -Quemaduras, trauma / dao del tejido destructivo -Necrosis -Infartos/isquemia/gangrena/vasculitis -Hemlisis -Malignidad -Leucemia mielgena crnica (LMC) -Leucemia linfoctica crnica (LLC) -Carcinomatosis/cancers (necrosis de los tejidos)

**

*Los procesos inflamatorios* -Enfermedad vascular del colgeno

*Metablica* -Cetoacidosis diabtica -Gota

*Txicas o drogas* -Esteroides (aguda o crnica), litio, uso intenso de tabaco, otros**

*Cmo se puede distinguir las reacciones leucemoide / leucocitosis neutroflica grave (alta CGB + / Plaquetas altas) de la LMC?*

**

*Leucemia mieloide crnica leucemoide*

Reaccin

Esplenomegalia No esplenomegalia Fosfatasa alcalina leucocitaria baja Fosfatasa alcalina leucocitaria ALTA Acido urico alto Acido rico normal T9, 22 (el cromosoma Filadelfia) bcr / abl genee

**

*Cules son las tres principales causas de una protena total elevada?** * VIH TB Mieloma mltiple

*Qu medicamentos comunes pueden estar asociados con adenopatas inexplicables?** *

*Alopurinol* Atenolol Penicilinas Trimetoprim / sulfametoxazol Captopril Carbamazepina Hidralazina Fenitona Pirimetamina Oro**

*Qu puede dar un hemocultivo falso + para la Bactec T /sistema de alerta?*

Leucocitosis Glbulos rojos Coincidiendo el uso de antibiticos por parte del paciente

Por qu? Las bacterias que crecen en el medio producen CO2. Una vez que los niveles son suficientemente altos, el CO2 atraviesa la membrana semipermeable en la base de la botella Bactec y produce un cambio de color en el sensor colorimtrico de CO2. El cambio es detectado por un escner de colorimetra y la botella es marcada. El micro tech luego realiza una tincin de Gram o tincin de naranja acridina para determinar si las bacterias estn presentes, as como una subcultivo para medios slidos. El cultivo es reportado como + solo si la tincion y / o el cultivo son +. Los glbulos rojos pueden producir algo de CO2 de

fondo, igual que altos que los globulos blancos. (Estreptococcus pneumoniae a veces puede dar un falso negativo. En el cultivo, Etreptococcus pneumoniae puede lisar los glbulos rojos, as como en s mismo (achocolatando el medio), mientras produce CO2 y marca +. Las tinciones, y posiblemente, el cultivo, puede ser negativo debido a la lisis que puede parecer una falsa alarma + para Bactec, pero en realidad ser un falso por cultivo y tincin).**

*Adems de leucocitosis, cmo puede un conteo sanguneo completo proporcionar pistas para un absceso sin drenaje o un absceso en desarrollo (flemn)?*

Trombocitosis reactiva**

*Causas no infecciosas (5) de los infiltrados pulmonares agudos:*

Aspiracin ICC Embolismo Pulmonar Hemorragia SDRA

*Lista de las causas de una monocitosis:*

Endocarditis bacteriana subaguda Tuberculosis diseminada Fiebre tifoidea Malaria Leishmaniasis visceral (kala azar) Enfermedades vascular del colgeno Sndromes mielodisplsicos / malignidad Helicobacter pylori

*Causas de una linfocitosis:*

VEB, CMV, VVZ, otros virus Toxoplasmosis Tos ferina, tuberculosis LLA, LLC Inmunizacin, enfermedades autoinmunes, rechazo del injerto Hipotiroidismo Linfocitosis relativa asociada con granulocitopenia

*Lista 5 reactantes de fase aguda (aumento de la infeccin, traumatismo, inflamacin, o malignidad):*

Protena C-reactiva VSG Ferritina Haptoglobina CGB, especialmente las formas inmaduras / bandas (desviacin a la izquierda) Plaquetas

**

*Causas de trombocitosis:*

Deficiencia de Fe + + Reactiva -Inflamacin -Infeccin (abscesos no drenados, especialmente absceso esplnico, tuberculosis) -Rechazo Trastornos mieloproliferativos / sndrome mielodisplsico

*Lista de 4 condiciones no infecciosas que deben ser consideradas en la investigacin de Sndrome de Fatiga Crnica.*

Hipotiroidismo Anemia Depresin

Trastornos reumatolgicos (por ejemplo, artritis reumatoide, fibrositis / fibromialgia/ lupus eritematoso sistmico)

*Nombre tres caractersticas de cada una de las siguientes enfermedades:*

Artritis reumatoidea Serositis, Ndulos, Vasculitis. Granulomatosis de Wegener Sinusitis, Bronquitis, Nefritis Sndrome de Behcet lceras genital y oral, Uvetis, Meningitis asptica Amiloidosis Macroglosia, Nefropata (proteinuria), Neuropata Perifrica Sndrome de Sjgren- Queratoconjuntivitis seca, Xerostoma, Parotiditis -

*Nombre de los 5 componentes del Sndrome de CREET.** * C Condrocalcinosis R Sndrome de Raynaud E alteracin de la motilidad del esfago E Esclerodermia T Telangiectasia

*Qu articulaciones de la mano hacen el efecto siguiente?*

Psoriasis- Distal InterFalangica Artritis Reumatoidea Proximal InterFalangica, MetaCarPofalangicas Hemocromatosis MCP del 1er y 2do dedos

Osteoartritis todas

*Dnde estn los nodos de Heberdens?*

Estn ubicados en la cara dorsolateral y medial de la articulacin interfalangica distal (IFD) de los dedos. Estn asociados con la osteoartritis

*Dnde se encuentran ndulos de Bouchard?*

En las articulaciones Proximales Inter Falangica. Tambin con la osteoartritis.

*Definir los trastornos mieloproliferativos y nombrar 6:*

Def: Situacin clnica que resulta de la produccin incontrolada de todos los elementos de la mdula sea. *P *- Policitemia vera - incremento de GR *T* Trombocitosis esencial *M* Metaplasia Mieloide Idiopatica (pancitopenia), leucemia mielgena aguda (LMA) *L* LMC inc. CMB

*E *- Eritroleucemia

Esplenomegalia resultados de la hematopoyesis extramedular

*Los tumores asociados con VEB:*

Linfoma de Burkitt Carcinoma nasofarngeo Linfoma de clulas B en pacientes inmunosuprimidos.

*Nombre de los 6 tumores que suelen metastatizar a hueso:*

* *Rin La tiroides Pulmn Prstata Testculos Mama

*Dar los 11 criterios revisados para la clasificacin del LES:***

*SUAF SRAINEE:*

*S* serositis pleuritis, pericarditis *U* lceras orales ulceracin nasofarngea, generalmente sin dolor *A *- Artritis no erosiva con dos o ms articulaciones perifricas sensibilidad, inflamacin, derrame. *F* Fotosensibilidad luz del sol erupcin *S* Trastornos de la sangre - anemia hemoltica, leucopenia, linfopenia, o trombocitopenia. *R* Renal proteinuria o cilindros celulares *A *- ANA (anticuerpos antinucleares) *I *- Trastornos inmunolgicos: + LE preparacin de clulas o anti-ADN de doble cadena; falsa RPR + *N *- Trastornos Neurolgicos *E* Erupcin malar - eritema fijo en las eminencias malares, repuestos pliegues nasolabiales. *E* Erupcin discoide eritematosas, placas elevadas con descamacin y taponamiento folicular; cicatrices atrficas puede ocurrir con lesiones viejas.

Lupus Discoide inducido por frmacos de recambio renal.

*La artritis reumatoide es una enfermedad sistmica que puede presentarse con fiebre durante los brotes.** Cules son los tres componentes de esta enfermedad? *

Ndulos en superficies de extensin Serositis Vasculitis

*Nombre de los agentes quimioterapicos asociados con los siguientes efectos secundarios:*

Hepatotoxicidad

*M* - Metotrexate (MTX) *A *- Asparaginasa

*M* - Mercaptopurina *A *- Adriamicina

Cistitis hemorrgica Ciclofosfamida Neurotoxicidad Metotrexato, Vincristina, Vinblastina Cardiotoxicidad Adriamicina, doxo-y daunorrubicina

Nefrotoxicidad -

*P* Platimum *M* MTX *E* Estreptozocina

Fludarabina Inmunodeficiencia Mediada por clulas (especialmente con

esteroides)

*Qu sndromes se asocian con el ALH (Antigeno Leucocitario Humano) B-27?*

*E* Espondilitis anquilosante / enfermedad inflamatoria intestinal espondilitis *U *- Uvetis anterior *R* Sndrome de Reiter *A* Artritis psorisica

*Qu sndromes estan asociados con los tipos D del Antigeno Leucocitario Humano (ALH)?*

El sndrome de Sjogren Miastenia gravis La enfermedad de Addison Enfermedad celaca Hepatitis crnica DMID Tirotoxicosis la enfermedad de Hogkin Esclerosis mltiple LES

*Nombre 5 tipos de citoquinas*

Interferones (IFN) Interleukinas (IL) Factores de crecimiento (FC) Factor de necrosis tumoral (FNT) Factor estimulante de colonias (FSC)

*Nombre tres tipos de interfern, una fuente importante, y la accin.*

*TIPO ACCIN*

FUENTE

IFN-alfa Leucocitos antiviral / via antitumoral

macrofagos y clulas NK

IFN-beta la misma

Fibroblastos

IFN-gamma T

Clulas

antiviral / antitumoral e

inmunoregulatoria, via especifica

de macrofagos.

*Describir la accin de las 6 interleuquinas. (LETS BBB) * IT-1 Factor de activacin de linfocitos (clulas T) IL-2 Factor de crecimiento de las clulas T IL-3 Factor de crecimiento de clulas madres (factor estimulante de colonias) IL-4 Factor de crecimiento de clulas B IL-5 Factor de diferenciacin de clulas B IL-6 Factor de Maduracin de clulas B

*Qu interleukina no se produce principalmente por las clulas T?** * IL-1 es producida por los macrfagos.

*La produccin de interleucina que est directamente bloqueado por ciclosporina?***

IL-2

*Nombre 5 tipos de factores de crecimiento.*

*PDGF* factor de crecimiento derivado de Plaquetas *FGF* Factor de Crecimiento de Fibroblastos *NGF* factor de crecimiento del Nervio *FEAG* factor de crecimiento epitelial *TGF *factor de crecimiento tumoral beta

*Factores estimulantes de colonias que intensifican el crecimiento y diferenciacin de las clulas madres de mdula sea.** **Nombre de los 5 tipos de factores estimulantes de colonias, y las clulas madre que ellos afectan.*

*G-CSF* Granulocitos *M-CSF* Macrfagos *GM-CSF* Granulocitos / Macrfago *IL-3* clulas madres pluripotenciales Eritropoyetina clulas madres eritroides

* *

*INMUNODEPRIMIDOS Y PACIENTES CON CANCER***

**

*Cul es la causa ms comn de la inmunodeficiencia global?*

Desnutricin. Predispocision a la gastroenteritis y a la neumona.

*Cules son los tipos ms comunes de inmunodeficiencia en adultos?*

Deficiencia y defectos de anticuerpos Deficiencia del Complemento

**

*Lista de pistas infecciosas y no infecciosas para la inmunodeficiencia en los adultos:** ** Infecciosas*

Infecciones frecuentes Infecciones de gravedad inusual Infecciones refractarias o prolongadas Patgenos inusuales

Historia familiar de autoinmunidad o malignidad

*No infecciosas*

Mala cicatrizacin de las heridas Mala denticin Causa de bronquiectasias no determinada

*Los patrones de infeccin, inmunodeficiencia, y patgenos de inters:*

/Sinusitis recurrente, infecciones pulmonares/ Meningitis Bacteriemia Pensar en defecto de inmunoglobulina o del complemento Preocuparse por neumococos Haemophilus influenzae Neisseria meningitidis Campylobacter Giardia

/Abscesos recurrentes no relacionados con las zonas apocrinas / cuerpos extraos / colonizacin por Staphyloccocus aureus/ Pensar en defecto de neutrfilos Preocuparse por Stafilococos aureus

Bacilos Gram negativos Aspergillus Nocardia

/Hongos y virus oportunistas/ Pensar en inmunodeficiencia de clulas T/ mediada por clulas Preocuparse por Candida Criptococcus CMV, VHS Micobacterias

/Eccema, levaduras mucocutneas, piel/infecciones de tejidos blandos, infecciones del tracto respiratorio/ Sndrome de Job / hiperinmunoglobulina E Buscar por IGE> 2000 UI / ml

/Anemia hemoltica con habas (favismo), a las sulfonamidas/ Neumona recurrente, infeccin severa de tejidos blandos y de piel, Pensar en deficiencia severa de G6PD General: actividad <5-10% de de G6PD Confiere cierta inmunidad a la malaria (como la enfermedad de clulas falciformes) por el aumento de la remocin de las clulas infectadas por el bazo **

*Furunculosis recurrente por Staphyloccocus aureus*

Buscar reas de colonizacin por S. aureus que no sean las fosas nasales y las zonas apocrinas: eccema no tratado, psoriasis, piel crnicamente anormales tales como el eczema dishidrotico / ponflix **

*Lista de los 3 tipos de deficiencias de clulas B*.

Deficiencia de IgA (Inmunodeficiencia mas comn en los pases desarrolladas.) Hipogammaglobulinemia de Bruton Inmunodeficiencia Variable Comn (IDVC)

*Qu enfermedades se asocian con la deficiencia de IgA y DIVC (Deficiencia Inmune Variable Comun)?*

Sinusitis, otitis y bronquitis (gripe por Haemophylus, neumococo).

*Nombre 5 tipos de deficiencia de clulas* T.

Sndrome de DiGeorge (ttano neonatal y la hipoplasia del timo) Candidiasis mucocutnea crnica (profilaxis con ketoconazol) EICS (Enfermedad de Inmunodeficiencia combinada severa)

Sndrome de Wiskott-Aldrich Ataxia-telangiectasia

*Cules son los tres tipos de SCID (Enfermedad de Inmunudeficiencia Combinada Grave)?*

Deficiencia de adenosina deaminasa (Tx: una unidad de sangre en el primer da de vida) Disgenesia reticular (disminucin de los PMN y las clulas T) Inmunodeficiencia tipo suizo (no clulas B o T)

*Cules son las caractersticas del Sindrome Wiskott-Aldrich?*

Es un *HITE* que se encuentra en los hombres: *H *- Hombres el exceso de sangrado del cordn umbilical o el sitio de la circuncisin! *I *- Inmunodeficiencia *T *- Trombocitopenia *E* Eczema

*Q**ue anuncia la presencia del trastorno de la adhesin de leucocitos?*

Infeccin no purulenta del cordn umbilical y la separacin tarda del cordn umbilical.

*Cules son las caractersticas y el tratamiento de la Telangiectasia Ataxia?*

* *Caractersticas: ataxia, telangiectasias, y la degeneracin del hgado Tratamiento: Gammaglobulina

**

*Cules son los tipos de defectos inmunolgicos no especficos?*

Defectos de los PMN =*QOIM* *Q* quimiotactismo como en el sndrome del leucocito perezoso *O* Opsonizacin (neumococo, Haemophilus) *I *- Ingestin *M* Muerte intracelular como en el sndrome de Chediak-Higashi y la enfermedad granulomatosa crnica. Se caracteriza por inclusiones intracelulares y abscesos de staphyloccocus en el cerebro, el hgado y los pulmones.

*Qu infecciones sugieren un defecto del complemento terminal?*

Crnica / gonococcemias recurrentes, o meningococcemias en adultos Es decir si> 1 episodio de Neisseria en la sangre, comprueba si hay defecto del complemento terminal.

*Cules son algunas de las causas secundarias de deficiencia inmune en adultos que no son causadas por el VIH, las drogas, o la esplenectoma?** **Y cul es el defecto, si se conoce?* **

*Diabetes mellitus* (hiperglicemia provoca disminucin de la quimiotaxis y la fagocitosis de los PMN, disminucin de la opsonizacin por el complemento, isquemia vascular) *Cirrosis *(aumento de glucocorticoides endgenos deficiente actividad de las clulas asesinas naturales, disminucin de la funcin / nivel de complemento sepsis, peritonitis) *El sndrome nefrtico* (prdidas urinarias / hipogammaglobulinemia peritonitis comn) *Hemodilisis /* *uremia* (mecanismo no conocido pero disminuyen clulas T / actividad de los PMN) *La dilisis peritoneal* (IG y complemento son removidos con la dilisis reduce la funcin de los PMN, peritonitis) *Enfermedades autoinmunes** *Vena safena cosecha / estasis venosa / linfedema crnico (agrupados linfticos = medio de cultivo =) *Tipos de cncer*

*Malnutricin proteico* *calrica* (disminucin de la fagocitosis, de la funcin de clulas T, funcin disminuida de anticuerpos especfica) *Traumas/quemaduras* (necrosis libera grandes cantidades de factor de necrosis tumoral /IL-1, aadir la prdida de la dermis con quemaduras)

*E**nfermedades infecciosas asociadas con defectos inmunolgicos:*

*Sarampin (*macrfagos / disfuncin de las clulas presentadoras de antgeno) Bacilos Gram negativos neumona

*Citomegalovirus* (disfuncin aguda de clulas T, produccin reducida de gamma globulina)

*Bacterias productoras de superantgenos Staph.aureus, estreptococos del grupo A* (Anergia de clulas T despus de la tormenta de citoquinas / respuesta inflamatoria sistmica

*Micobacterias* (organismo desactiva los monocitos que infecta)

*Protozoos* *tripanosomas, leishmaniasis, malaria* * (macrfagos / disfuncin de las clulas presentadoras de antgenos) * Malaria induce la disfuncin de las clulas T citotxicas en las clulas infectadas con EBV predispone al linfoma de Burkitt asociado al VEB

*Neoplasias hematolgicas asociadas con riesgos de infeccin incrementado:***

*Leucemia linfoctica crnica (LLC)* -Hipogammaglobulinemia, neutropenia, la reduccin de CD4 -Dar IGIV si los niveles de IgG estn bajos -En riesgo por PCP, Listeria -Aadir profilaxis contra PCP si tambin en la fludarabina (CD4 notablemente bajo)

*Leucemia linfoctica aguda (LLA)* -Esteroides riesgo de PCP Anadir al tratamiento durante la

profilaxis PCP

*Linfoma no Hodgkin (LNH) de grado intermedio-alto ** *Neutropenia relacionada con el tratamiento* *-Fludarabina riesgo por PCP Anadir al tratamiento durante la profilaxis PCP

*Mieloma mltiple (MM)* -Hipogammaglobulinemia (<20% de lo normal, pueden necesitar reemplazo de IgG), respuesta humoral reducida al contacto con antgenos, deficiencia inmunes mediadas por clulas - Sepsis / pneumococos Mayor riesgo de infeccin en el 1er y 2do mes de quimioterapia

*Leucemia Mieloctica Aguda(LMA)* -Altas dosis de Ara-C (HiDAC) asociada con / mucositis GI sustancial -Esperamos fiebre durante la neutropenia, sepsis GNR

*Leucemia Mieloide Crnica (LMC)* -Hipogammaglobulinemia, deficiencia inmune mediada por clulas

*Sndrome Mielodisplsico (SMD)* Neutropenia

*Leucemia de Clulas Pilosas (LCP)* 40% tienen menos de 500 clulas / mm 3 recuento de neutrfilos - Destruccin intracelular pobres -Infecciones pigenas son comunes**

**

*Trastornos linfoproliferativos con neutropenia funcional (neutrofilos con mal funcionamiento a pesar de un contaje de globulos blancos normal alto):*

*LMA *- capaz de fagocitar pero muerte intracelular deficiente *LLA *- muerte intracelular pobres *LLC* neutropenia leve-moderada *LMC* fagocitosis pobres, quimiotaxis pobre, crisis blstica (alta numero de WBC) Leucemia de clulas pilosas muerte intracelular pobre en la parte superior de la neutropenia absoluta

**

*Definir la neutropenia, y describir una lnea de tiempo para las infecciones preocupantes.*

Recuento absoluto de neutrfilos (ANC) = total de glbulos blancos x (% de neutrfilos + % de bandas) por ejemplo ANC = 7.000 clulas / mm 3 x (30% de PMN + 5% de blastos) = 7,000 x 0,35 = 2,450 Neutropenia = ANC menos de 1.500 clulas / mm 3

<http://www.gompfsidpearls.net/wp-content/uploads/2011/11/NeutrophilsInfx.png>[Clsico de referencia: Bodey GP et al. relaciones cuantitativas entre los leucocitos circulantes y la infeccin en pacientes con leucemia aguda. Ann Int Med 1966; 64:328-40] Menos de 500 clulas / mm 3 = mayor riesgo de infeccin grave empieza a subir. RAN <1000/mm3 = 20% de infeccin RAN <500/mm3 => 35% de infeccin RAN <100/mm3 => 50% de infeccin

<http://www.gompfsidpearls.net/wp-content/uploads/2011/11/Neutropenia-Duration-infx.png>

[Bodey GP et al. Ann Int Med 1966; 64:328-40] El riesgo de infeccin aumenta con la duracin y la profundidad de la neutropenia. neutropenia severa (RAN <100/mm3) durante 3 semanas = 100% el riesgo de infeccin**

*Definir los perodos de riesgo de infeccin durante la neutropenia febril, y las fuentes ms probables / sitios / tipos de infecciones observados en cada uno.** *

*Da 0-7*

**-Piel, translocacin intestinal, iletis terminal / tiflitis Gram neg, anaerobios

-Piel, translocacin intestinal, iletis terminal / tiflitis Gram neg, anaerobios

-Con una cobertura anti-Pseudomonal/GNR S. aureus, Viridans estreptococos, Corynebacterium

*Da 14/07*

-Translocacin Intestinal, punto de partida relacionada con la bacteriemia GNR, GPC (SAMR/EVR), levaduras (especialmente en los

antibiticos de amplio espectro), anaerobios **

*Da 14 +* *(neutropenia prolongada)*

-Intestino, piel, asociadas al hospital (pulmn / sangre / ITU) GNR (incluyendo multirresistentes), GPC (incluyendo estafilococos coagulasa negativos, SAMR / EVR), GPR (Bacillus, Corynebacterium JK), levaduras

-boca/dentadura anaerobios orales (Clostridia, Capnocytophagia, Fusobacterium)

-pulmones/Senos paranasales (exposicin ambiental, especialmente durante la construccin / remodelacin en el hospital) mohos

-reactivacin de herpes simple, varicela / herpes zoster, citomegalovirus, adenovirus, virus BK**

*Lista de los 5 organismos oportunistas que a menudo causan infeccin pulmonar en pacientes inmunocomprometidos. Que coloracion es usada histolgicamente * **

*C-PLANT* *C* Citomegalovirus H & E *P *Pneumocystis carinii Gomorri plata metenamina

*L *Legionella pneumophila plata Dieterle *A* Aspergillus plata metenamina Gomorri *N* Nocardia Acido resistente modificado *T* Toxoplasma gondii inmunoperoxidasa

**

*Nombre los tres perodos post trasplantes hematolgicos/ rganos slidos y las infecciones asociadas:*

*Perodo Temprano* (0-30 das) Infecciones Nosocomiales

*PERIODO INMUNOSUPRIMIDOS* (30-180 das) Infecciones Oportunistas Infecciones PLANT SNC Listeria / Cryptococcus FOD (Fiebre de Origen Desconocido) / GI-CMV

*Perodo Tardo* (180 o ms das) Infecciones de la Comunidad

*Nombre de las infecciones y condiciones de inters en los receptores de trasplante de mdula sea alognicos.*

*P**erodo de post-transplante (clulas de mdula nuevas siendo

producidas y no ms neutropenia)* - Persistencia de la deficiencia inmune de mediadores celulares -Enfermedad de Injerto contra Husped (esteroides y CyA reducir la inmunidad) -Moldes sinopulmonares siguen siendo un riesgo indefinidamente -CMV, herpes, varicela aciclovir / profilaxis con ganciclovir, PCR para CMV semanal -BK virus cistitis hemorrgica -HHV6 -Neumona viral por virus respiratorio de la comunidad (VSR, influenza, parainfluenza, adenovirus, rinovirus, metapneumovirus humano) -Hemorragia alveolar difusa

*Qu infecciones, injurias inmunolgicas, y toxicidades estn asociadas con cuales agentes inmunosupresores?***

La *c*iclosporina (CyA) / tacrolimus / mediada por *c*lulas / *C*MV, neumonas bacterianas -Insuficiencia renal -Temblor -Hepatotoxicidad

Azatioprina (Imuran / mediadores *c*lulares) -Pancitopenias -pancreatitis/hepatitis -Cncer de piel

Micofenolato mofetilo (*C*ellcept) / mediadores *c*lulares / *C*MV Diarrea

*C*iclofosfamida (Cytoxan) / clorambucil / mediadadores clulares -*Cistitis* hemorrgica -Supresin de la mdula sea

El*metotrexate* / supresin linfoide leve / esteroides concomitante (especialmente con enfermedades reumtica) -*PCP, CMV, hongos, Nocardia* -Hepatitis

*TNF*-alfa bloqueantes (etanercept, infliximab) / *mediada por clulas* T*uberculosis*, Histoplasma, PCP (Pneumocistis pneumonia (carini)) / Aspergillus / Coccidioides / Listeria Anlogos de purina (*fludarabina,* cladribina, pentostat) / limfopenia/*supresin del CD4 *

-*SA, GNRs*, el *PCP.* Listeria, diseminada*VZV*, CMV, Legionella, Nocardia Rituximab (*Campath*) / alemtuzumab / IgG y mediadores celulares

-*CMV (hemorragia alveolar / neumonitis hemorrgica*), el VHS (virus del herpes zoster), el PCP, Aspergillus Mucor, Cryptococcus, infecciones de la piel / otitis media / herpes hasta por 12 meses ms tarde Todos: postrasplante asociado con linfomas de clulas B (Virus del Eistein Barr)

Nivel de las terapias antitimoctica e inmunosupresin especifica, OKT3 -Seronegatividad para el VEB

**

*Cules son las interacciones antimicrobianas importantes con CyA?***

Eleva CyA (RECHAZO PRECIPITADOS!) antifngicos azoles isoniazida recordar Rechazo rifampicina Eritromicina / claritromicina

Reduce CyA

Rifampicina,

*Qu enfermedades produce el virus BK?*

Cistitis hemorrgica Estenosis uretral Disfuncion Heptica Especialmente en en la mdula sea / pacientes neutropnicos.

**

*Cules son los tipos de reactivacin de virus BK y JC?***

BK 50%

JC 5%

**

*Nombre dos tipos de enfermedad VEB despus del trasplante y tiempo de ocurrencia.***

Jvenes: 9 meses enfermedad viral Edad: 6 aos masa tumoral

*Nombre cinco de los tipos principales de agentes inmunosupresores utilizados en trasplantes:*

**

*Corticosteroides* accin ms amplia (bloquea la liberacin de Interleuquina I-V, la disminucin gamma IFN-, disminuye la produccin de Anticuerpos (Ac), y una respuesta inflamatoria contundente por estabilizacin de la membrana. *Citotoxinas* Azatioprina anlogo de la purina que interrumpe la sntesis del ADN Ciclofosfamida. *Anticuerpos antilinfocticos* OKT3 CD3 clulas T ciegas receptor FK506 assocd con linfoma postrasplante. *Ciclosporina *- inhibe la proliferacin de las clulas T -4 ayudadoras (bloqueo de Interleuquina-S). *Radioterapia*

*Lista de diagnsticos diferenciales por causas no infecciosas de infiltrados pulmonares en pacientes con cncer.*

Insuficiencia cardaca congestiva / sobrecarga de lquidos (Enfermedad Arteriosclerotica Coronaria, Interleukin-2 cardiotoxicidad directa) Edema pulmonar no cardiognico (Sindrome de Distres Respiratorio del Adulto, arabinsido de citosina, IL-2) Embolismo pulmonar (sndrome de hipercoagulabilidad) Neoplasia (metstasis, diseminacin linftica, infiltrados leucmicos con leucocitos> 100.000 clulas/mm3 en ALD / LMC) Leucemia Mielocitica Aguda/ Leucemia Mielocitica Cronica

Hemorragia alveolar difusa (autlogo / trasplante alognico de mdula sea) disnea sbita, tos seca (sin sangre), fiebre, sangre en BAL dar esteroides BONO (post-trasplante de MO, especialmente con crnica rechazo del transplante) esteroides Relacionados con las drogas (busulfn, bleomicina, BCNU, metotrexato, con menos frecuencia: ciclofosfamida, mitomicina , 6-mercaptopurina, semustina, etopsido vinblastina)

**

*Qu tipos de infeccin estan asociadas con el cncer cerebral?*

Infecciones postoperatorias, infecciones de la herida quirrgica, subgaleal / absceso subdural, meningitis

*Cules son los factores de riesgo para la infeccin postoperatoria en el cncer cerebral?***

Fuga de LCR

Antes de la neurociruga Quimioterapia Radioterapia (tambin radionecrosis, que pueden desarrollar osteomielitis) Esteroides prolongado infeccin de la herida quirrgica previa

Tiempo prolongado Insercin de un Implante Gliadel de quimioterapia en la cavidad tumoral

*Qu organismos causan infeccin en el cncer cerebral?*

E. aureus, GNR, Propionobacterium acnes, estafilococos coagulasa negativos, estreptococos anaerobios (especialmente despus de la ciruga del seno)

*Qu infecciones estan asociadas con cnceres ORL?*

Reactivacin del herpes, aftas Post-Operatorio la saliva contiene 10^8 UFC / ml (Unidades Formadoras de Colonia) (comparado a 105 UFC / ml de tejidos blandos infectados) 10-20% de tasa de infeccin de heridas Estreptococcus aureus, GNR, Candida, estreptococos anaerobios, herpes simple

*Qu factores de riesgo estn asociados con la infeccin en los cnceres del area ORL?*

* *Xerostoma Gingivitis, enfermedad dental

Mucositis con quimioterapia / TR Tiempo prolongado Transfusion sangunea peri operatoria Redconstrucciones de Colgajos Radionecrosis de la mandibula polimicrobiana, osteomielitis

*Qu infecciones estan asociadas con cnceres de pulmn?*

Neumona, en cualquier momento, especialmente obstructiva Fstula Broncho-pleural (FBP) / empiema post-Operatorio Infeccin de la herida quirrgica post-operatoria S. aureus / MREA (Metacilina Resistente a Estafilococcus Aureus), GNR (Gentamicina Resistente), estreptococos alfa, Candida (empiema por Fistula Bronco-Pleural FBP no, neumona), Legionella, micobacterias no tuberculosis

*Qu infecciones pueden estar asociadas con el cncer de colon?*

Post-Operatorio: infeccin de la herida quirrgica perforacin / abscesos intra-abdominales (tambin pre-OP y con colitis por radiacin) infecciones de malla fstulas enterocutneas (tambin con colitis por radiacin)

Con resecciones abdomino-perineal: abscesos perineales, abscesos pre-sacra, osteomielitis sacro-ilaca S. aureus, GNR, Enterococcus / Vancomicina Resistente (VRE), Candida

*Cules son los factores de riesgo para la infeccin en el cncer de mama?*

La herida quirrgica / drenajes Del implante del pecho / expansor Seroma Post-operatorio colgajos miocutneos Fumadores colgajo de necrosis / infeccin Linfedema Antes de la piel, infeccin de tejidos blandos TR / radiacin

Estafilococcus Aureus / Metacilina Resistente a Estafiloccous Aureus, estreptococos beta, GNR, micobacterias de crecimiento rpido

*Qu causa infecciones de tejidos blandos / piel en pacientes con cncer?*

Deterioro de drenaje linftico, edema -Cirugas, invasin tumoral, diseccin de ganglios linfticos Estreptococos del grupo A, SA (estafilococcus aureus) / MRSA (Meticilina Resistente a Estafilococcus Aureus), GNR (Gentamicina Resistente),

polimicrobiana Infecciones de heridas -Cirugas Reducido si cualquier TR se retrasa 3-4 semanas despus del postoperatorio

*Cules son las fuentes de infeccin en la bacteriemia asociada a catter?*

Piel>> punta del catter > hematgenos > infusiones = lavarse las manos:}

*Cul es la complicacin infecciosa ms importante relacionada con la radioterapia (RT) a la cabeza, los huesos largos y la columna vertebral?*

Radionecrosis w / celulitis resultante y posiblemente osteomielitis (infeccin del hueso muerto) Principal flora local Huesos largos/ columna piel / organismos hematgenos

*Cul es la complicacin infecciosa ms importante relacionada con la radioterapia (RT) en el trax?*

Esofagitis por radiacin herpes simple superpuestas, citomegalovirus, levaduras

Dao alveolar por radiacin neumonitis por radiacin / fibrosis / pleuritis / (Bronquiolitis obliterante con neumona en organizacion) BONO patgenos respiratorios adquiridas en el hospital y en la comunidad, Aspergillus (especialmente con ndulos / cavidades) Neumona recurrente? Pensar en la radiacin fstula traqueo-esofgica con aspiracin!

**

*Tiene la radioterapia (RT) efectos asociados a la inmunidad distintos del dao tisular local?** *

S. Hay una pequea cada en la quimiotaxis de los neutrfilos durante 3 das despus de la RT A principios de RT = muerte de las clulas epiteliales (mucositis, xerostoma, proctitis) Ms tarde (6 meses +) = reduccin de los fibroblastos, mala cicatrizacin, fibrosis, vascularizacin reducida**

*Qu infecciones pueden aumentar en el lupus eritematoso y por qu?*

Bacteriemia por Salmonella Herpes CMV

Parvovirus B 19 Neumococo Nocardia (esteroides)

*A causa de:* Deficiencia del Complemento Linfopenia CD4 Asplenia funcional Esteroides

*Qu patgeno puede superinfect en la granulomatosis de Wegener?*

Staphylococcus aureus

*Lista de 4 agentes infecciosos a los cuales los pacientes esplenectomizados son susceptibles:*

* **BEDS* *B* Babesiosis *E*- organismos encapsulados (neumococo, Haemophilus, Klebsiella) *D* DF-2, ahora canimorsus Capnocytophaga *S* Salmonella**

* *

*ZOONOSIS e INSECTOS INFECCIONES ASOCIADAS *

**

*Asociaciones clave de los animales, patgenos, presentacin clave, y Droga de eleccin:***

*Cabras / queso de cabra*-Brucelosis-absceso esplnico-Doxiciclina + Gentamicina

*Conejos/Conejos desollados-*tularemia-linfadenopatia- Esteptomicina

*Exposicin a la placenta / animales parturientas/ animales con pezuas*Fiebre Q / Coxiella burnetii-pneumonia/esplenomegalia-Doxyiciclina

*Perros / bovinos / ratas (orina) en contacto con agua dulce*-Leptospirosis-conjunctivitis/hepatitis/renal insuficiencia G-PCN

*Felino> mordeduras caninas*- Pasturella multocida-celulitis/osteomielitis/sepsis-PCN G

*Felinos*-Bartonella henselae lesiones angiomatosas en SIDA Doxiciclina o Azitromicina

*Mordeduras de perro* / lamidos Capnocytophagia

canimorsis-Amoxicilina/clavulanate o Clindamicina Pasturella-Penicilina-G

*Reptiles, tortugas*-Aeromonas (mordeduras), Salmonella (slo estar cerca de un reptil es un factor de riesgo para estos ltimos, debido a las heces transportadas) -celulitis/bacteremia-Quinolona

*Loros / guacamayos / tienda de animales *psittacosis-pneumonia/esplenomegalia-Doxiciclina

*Guinea cerdos *Salmonella- como el anterior

*Hmsters / roedores- * virus de la coriomeningitis linfoctica -meningitis

*Pezuas de Ganado- Bacillus anthracis* mediastino ensanchado/ shock sptico / lesiones de piel gelatinosas de color negro ciprofloxacina (+ Clindamicina si hay sepsis)

*Carne de cerdo / embutidos / carne de oso o puma ahumado -* Trichinella spiralis miositis, eosinofilia, edema periorbitario, baja velocidad de sedimentacin globular -albendazol/mebendazol

*Las ardillas voladoras-*Rickettsia prowazekii (tifus epidmico)

*Perros de la pradera / ratas gigantes de Gambia*, la viruela del simio-ACV**

*/Asociaciones claves de insectos, patgenos, presentacin clave, y

Dexoxicorticosterona:/*

*/

(* escara@sitio de la picadura/**//*

*Las garrapatas / actividad al aire libre:***

*Enfermedad de Lyme (Borrelia burgdorferi)* precoz eritema migratorio, neuritis craneal facial, neuropata perifrica, meningoencefalitis linfoctica, con menos frecuencia miopericarditis / bloqueo AV, artritis crnica) Doxiciclina / Ceftriaxona**

*Babesia microti- *enfermedad parecida a la gripe, anemia hemoltica / hepatoesplenomegalia / Noreste o Noroeste de los EE.UU., cruz de Malta en los glbulos rojos Clindamicina + Quinina o Atovacuona + Azithro

*Fiebre Moteada de las Montaas Rocosas (rickettsia R.)-* enfermedad similar a la gripe, erupcin petequial / trombocitopenia (~ meningococcemia) 3-5 das de enfermedad, edema de manos y pies / periorbitario-Doxiciclina

*Ehrliquiosis *enfermedad**similar a la gripe (Fiebre Moteada de las Montanas Rocosas pero sin el moteado, trombocitopenia, no hay edema enmanos / pies, mrula en la capa leucocitaria, Noreste o Noroeste de los EE.UU.)-Doxiciclina

**

** Fiebre fricana por mordedura de garrapata (R. africae) / frica

subsahariana-*escaras mltiples, fiebre, linfadenopata Doxiciclina-

** Fiebre botonosa / Fiebre Moteada del Mediterrneo (R. conorii) / N.** **frica*-pequenas escaras oscuras en el centro tache noire, fiebre, linfadenopata, Doxiciclina

*La fiebre Q (tambin a travs de aerosoles / exposicin de la placenta)-*Fiebre de Origen Desconocido, esplenomegalia, cultivo negativo, endocarditis- Doxiciclina

**Fiebre recurrente (Borrelia recurrentis / hermsii)-*escaras, una enfermedad similar a la gripe, 3-5 de recadas- Doxiciclina

*Tularemia (tambin aerosoles / sangre de conejo),* lcera en la picadura, linfadenopata, enfermedades como la fiebre tifoidea-, o pneumonia- Etreptomicetina/Gentamicina

*F**iebre por la garrapatas de* *Colorado* ~ Fiebre Moteada de las Montanas Rocosas, erupcin <, enfermedad bifsica, Oeste de los Estados Unidos Doxiciclina

**

*P**arlisis por garrapatas* como Guillaine-Barre (neurotoxina de la garrapata) que resuelve en unl plazo de 12-24 horas de la eliminacin de garrapatas- no requiere tratamiento mas que remover la garrapata del cuero cabelludo

*Pulgas / ratas:* *Yersinia pestis / Suroeste de los Unidos / ardillas* -pneumonia/buboes Estreptomicina / Doxiciclina *Tifus murino (Rickettsia mooseri)* similar a la gripe, erupcin cutnea, *infestaciones de ratas* Doxiciclina

**

*Ratones / caros:*

* *Richettsialpox (Rickettsia akari*)-escaras, leves similares a la gripe, erupcin maculopapular-vesicular, Ciudad de *Nueva York / parques de la ciudad* Doxiciclina

*N**iguas:* *Tifus de las malezas (tsusugamushi R*.), fiebre, gripe, erupcin cutnea, adenopata Doxiciclina

*Piojo del cuerpo / personas sin hogar / guerra:* *Bartonella quintana* la fiebre de trinchera, fiebre de las trincheras urbanas / endocarditis en personas sin hogar Doxiciclina *Tifus epidmico / ardillas voladoras / enfermedad de Brill Zinsser (R. prowazekii) -* similar a la gripe, erupcin cutnea, puede reaparecer aos ms tarde como la enfermedad de Brill-Zinsser Doxiciclina

*Mosca de la arena:* *Leishmania-*hepatosplenomegalia/fiebre (Amrica Latina / Oriente Medio), lceras cutneas (zonas tropicales) anfotericina B, drogas antimoniales / estibogluconato *Bartonelosis,* fiebre de la Oroya / verruga peruana Doxiciclina / Azithro

*Mosca Negra :* *Oncocerca vlvulus* Ceguera del rio - ivermectina

*Mosca tse tse / safari africano:*

* Trypanosoma brucei rhodesiense (frica Oriental)* fiebre alta repentina, mialgia, cefalea, chancro indoloro en la picadura, debilitamiento, coma, muerte con o sin pronto tratamiento)-*EMERGENCIA* suramina, pentamidina (melarsoprol) / arsnico si SNC *llame al Centro de Control de Enfermedades* *T. b.** **gambiense (frica Occidental),* ms *indolente* la enfermedad del sueo, ganglios retro cervicales prominentes (signo de Winterbottom), hepatoesplenomegalia, estado mental decado. Suramina, pentamidina

*Chinches Rodvidos / Amrica Latina:* *Enfermedad de Chaga (Trypanosoma cruzi *-en forma de C, el triponosoma en la sangre): aguda, signo de Romaa (edema periorbital), fiebre,

miocarditis, crnicamente, fiebre, hepatoesplenomegalia, acalasia / megacolon, miocardiopata nifurtimox, benzonidazol**

*Mosquitos / Amrica Latina / Asia / trpicos:** El dengue*-similar a la gripe, + / erupcin**

* *

*VIAJES Y EFERMEDADES ASOCIADAS GEOGRAFICAMENTE POR EL SINDROME*

**

*Infecciones communes por el period de incubacion:*

(Ryan ETet al. /N Engl J Med/ 2002;347:505-16)**

**

*Tiempo

Enfermedad*

*Menos de 2 semanas

Malaria*

* Dengue*

* Fiebre moteada (infeccin por ricketsia)*

* Tifus de las malezas*

* Leptospirosis*

* Fiebre tifoidea*

* VIH Aguda*

* Tripanosomiasis de Africa del Este (EMERGENCIA, das-semanas)*

* Campilobacteriosis, salmonelosis, shigelosis*

**

*2-6 semanas malaria

**Fiebre tifoidea

Hepatitis A

Hepatitis E

Esquistosomiasis aguda (fiebre de Katayama) Absceso heptico amibiano (Entamoeba histolytica) Leptospirosis HIV agudo Tripanosomiasis del African del Este (Emergencia, das- semana)

Fiebre hemorrgica viral Fiebre Q**

*Ms de 6 semanas*

Malaria Tuberculosis Leishmaniasis visceral Filariasis linftica Esquistosomiasis Absceso heptico amibiano Micosis crnica Hepatitis E Rabia Tripanosomiasis de frica Occidental (meses a aos)

**

*Qu ests buscando en un frotis de sangre en un viajero con fiebre?***

Malaria!!! Tripanosomiasis africana Fiebre recurrente / Borelia recurrente Bartonella Babesia

*Cules son los sntomas, signos y asociaciones de la brucelosis?***

FOD (Fiebre de Origen Desconocido) Fatiga crnica Osteomielitis-sacroiletis Hepatoesplenomegalia- absceso esplnico / infarto Epididymoorchitis Meningitis crnica

Leche no pasteurizada de cabras, carne cruda, las granjas

**

*Por qu debe tratar la fiebre moteada de las Montaas Rocosas dentro de 5 das?** *

La mortalidad se incrementa de un 6-7% a casi el 25% si el tratamiento se retrasa.**

*Diagnstico diferencial por sndrome:*

Enfermedades Tifoidea (fiebre, esplenomegalia, adenopatas, cefealeas) NO erupcin: Sudeste Asitico Melioidosis Amrica Latina / Costa mediterrnea / Golfo Prsico Brucelosis Per / Ecuador Bartonelosis Viedo de Nantucket / Martha s / NorEste de los EE.UU. Babesiosis Borreliosis no Lyme (vase adelante Fiebre Recurrente) Conejos Tularemia Amrica Latina / Oriente Medio Leishmaniasis visceral** * *

*Erupcion:*

***Paises en desarrollo/ India/ Asia * Fiebre Tifoidea/ Salmonelosis (manchas rosadas en el tronco)

*Fiebres moteadas (erupcin maculopapular, fiebre, trombocitopenia, todas transmitida por garrapatas, excepto Rickettsia akari y R. prowazeckii)*

*Las naciones en desarrollo / multitudes / guerra / refugiados*

*Rickettsia prowazeckii* - *Ardillas voladoras* -20 A 50 aos ms tarde, la enfermedad de Brill-Zinsser es recurrente, leves *Mediterrneo / Nor** **frica / Mar Negro*

Rickettsia conorii, fiebre botonosa / Fiebre moteada del Mediterrneo (FMM)

-70% Escara negra simple- pequeas ulceras **

*Japn*

Rickettsia japonica / Fiebre moteada japonesada (similar a fiebre moteada del mediterraneo FMM)

*Costa del Este de Australia*

Rikectsia australis Tifus de la

garrapata de Queensland *

*Sub-Sahara Africano*

*R. africae*- Fiebre por la Picadura de la Garrapata Africana -Comn en los viajeros / historia de los safari, leves - Escaras mltiples

*Ciudad de Nueva York / parques de la ciudad / ratones infectados caros del ratn***

*R. akari- Rickettsiosis* - Escaras n pequea costra, erupcin cutnea puede ser vesicular *Asia / Cuenca del Pacfico / Australia-niguas / matorral / turistas*

*tsutsugamushi Orentia-*Tifus de las malezas -Fiebre alta, Cefalea intensa -multiples escaras negras

*Sndromes parecidos a la gripe (fiebre, dolores)*

** SE / SC de EE.UU., los estados de roca Montaa / Cape Cod / Long Island-Primavera/Verano *R. rickettsias*

-Fiebre de las Montaas Rocosas (FMR) -Cefalea, mialgias, nuseas sndrome viral ~ - tobillos/munecas, edema de manos / pies / tronco -Erupcin 3-5dias mas tarde -trombocitopenia/shock *Nor-Este o Nor-Oeste de los EE.UU*. *Ehrlichiosis* -Igual que la FMR, excepto sin erupcin, no hay edema en manos / pies, mrula en la capa leucocitaria**

*Fiebre recurrente (fiebre que desaparece y vuelve)* *En Estados Unidos exposicin a garrapatas / durmiendo en una cabaa de troncos*

*Borrelia hermsii* recadas 4 o 5, neuritis craneal

* **La fiebre bifsica*

*Oeste de los EE.UU.* la fiebre por garrapatas de Colorado *Oriente Medio/ Amrica Latina/ Rwanda */ Los piojos/ refugios / guerra / epidemias - Borrelia recurrentis / mellitensis

*Enfermedad de Lyme*

*NE /US occidental garrapata Ixodes ninfa / hembra adulta***

*Borrelia burgdorferi* **

*Temprano / Etapa 1* = eritema migrans, la mordida posterior 3d-1Mo, migra / se desvanece 3-4 semanas **

*Diseminada / Etapa 2* = idioptica la parlisis de Bell, la carditis / bloqueo AV, la meningitis, artritis, 3d-6wks El sndrome de Bannwarth trada clsica de neuroborreliosis aguda: 1. meningitis linfocitaria 2. parlisis del nervio craneal 3. radiculoneuritis La ceguera puede ser visto, especialmente los nios, con aumento de la presin intracraneal o la neuropata. **

*Tardo / Etapa 3* = artritis, polineuropata, rara vez la encefalopata crnica; meses-aos Dx hallazgos clnicos, el aumento de 4 veces en la IgG, IgM +, o PCR sinovial (75-85% de sensibilidad)

*Fiebres hemorrgicas*

*SW EE.UU.* inhalacin de *orina de roedores desecada*, ratones de ciervos *Sin Nombre por hantavirus* Sindrome Respiratorio Agudo (SRA) despus de 4-5 dias, prdromo similar a la gripe,*leucocitosis*, coagulacin intravascular diseminada CID**

*Caribe y Asia y el trpico en todo el mundo los mosquitos***

* Dengue fiebre rompehuesos* Fiebre, erupcin cutnea similar a quemadura de sol, adenopatas, severas mialgias / artralgias -w enfermedad bifsica / segundo exantema, fiebre escasa de las palmas / plantas de los pies la infeccin repetida puede causar hemorrgia/ shock por sndrome febril, sangramiento / coagulacin intravascular diseminada, fascie y manos edematosas

*Fiebre Amarilla* -Infeccin subclnica ictrica con hemorragia heptica / renal / ictericia insuficiencia cardaca, hemorragia gastrointestinal (vmito negro) -WBC baja / PLT -IgM, aumento de 4 veces el valor de la IgG

*frica* *bola / Marburg-Zaire/Sudan, monos? / Murcilagos?* Enfermedad similar a la gripe -N/V/hemorragias difusas-diarrhea /muerte Neutrofilia / trombocitopenia grave

IgM/ aumento de la IgG en 4 veces su valor, DFA / microscopa electrnica de los tejidos, laboratorio de contencin cultivo de alto nivel (BSL-4) *Lassa / frica occidental *-Inhalacin de excretas de roedores *-Ribavirina*

**

*Congo*-garrapatas, trasmisin sanguinea/ de humano a humano *Fiebre hemorrgica* de *Crimea-Congo *(tambin Rusia y los Balcanes) *Del Valle del Rift *bovinos/ovejas/mosquitos**

*Amrica del Sur* *Fiebre Hemorragica Boliviana* (Producida por el Machupo virus)

-Inhalacin de excretas de roedores

* *

*MISCELLAN**EOS*

*Qu es la pasteurizacin y donde es ms importante por qu?*

El Emperador Napolen III le pidi al Dr. Luis Pasteur investigar las enfermedades que afligian a los viedos con prdidas econmicas a la industria del vino. Pasteur demostr que las enfermedades del vino son

causadas por microorganismos que pueden ser destruidos por el calentamiento del vino a 55 C durante varios minutos. Aplicada a la cerveza y a la leche, este proceso, llamado pasteurizacin, pronto se empez a usar en todo el mundo. Con pasteurizacin HTST (alta temperatura y tiempo corto), la leche cruda se calienta a un mnimo de 161 grados F durante 15 segundos, seguido de un enfriamiento inmediato. Este mtodo produce leche con una vida til de 14 a 17 das. Con ultrapasteurizacin, la leche se calienta a un mnimo de 280 grados F durante dos segundos y luego se refrigera inmediatamente. Este mtodo produce una vida til de alrededor de 60 das, cuando no es abierto y refrigerado. Patgenos muertos: M. bovis-uno del grupo del M. tuberculosis, que causa la tuberculosis humana y bovis a partir de la leche no pauterizada , Brucella, Escherichia coli, Listeria

*Qu potencial patgeno no es asesinado por la pasteurizacin?*

* *Mycobacterium paratuberculosis, una de las causas de la enfermedad de Johne en las vacas, que es muy similar a la enfermedad de Crohn, existe un vnculo?**

*Qu otros principios dilucidados por el Dr. Pasteur, nosotros utilizamos diariamente?*

La teora de los grmenes en 1857, demostraron que la putrefaccion / fermentacin ocurri slo en la exposicin a aire contaminado. Si los grmenes pueden causar la fermentacin, ellos podran bien causar enfermedades humanas y animales y ser transmitidas.

Teoria de la Vacuna el aadio a los trabajos de Jenner en vacunacin, y desarrollo las vacuans contra el ntrax y la rabia.

*Quin es el padre de la epidemiologa moderna?*

* *El Dr. John Snow, quien en 1854, despus de entrevistar a familiares de las vctimas, el localizo una epidemia de clera en la bomba de Broad Street en la cual la mayora haban tomado agua. Convenciendo a las autoridades para retirar el mango, se detuvo la epidemia durante dos semanas. Los trabajadores de una fbrica de cerveza adjacente, les distribuyeron cerveza gratis diariamente, y ellos no sufrieron de clera!. Una taberna aledaa ha sido nombrada despus de Snow, un hecho que le hubiera consternado, ya que aborreca el alcohol.**

*Quin es el padre de control de la infeccin?*

*Dr. Ignaz Semmelwiess Philippe, quien seal que los pacientes de los estudiantes de medicina se estaban muriendo de sepsis perpeural / Grp Un estreptococo despus del parto, mientras que los pacientes de las comadronas no lo hizo. Tomando nota de que los estudiantes de medicina realizaron autopsias (presumiblemente en las que murieron de sepsis!) Entre las entregas, el los haba lavado con cal clorada despus de las autopsias mort 20% a 1%. La profesin mdica se horroriz y le dio la espalda, y muri sin un centavo y demente en un manicomio. El sentido comn era tan poco apreciado en su tiempo.

*Quin es el padre de los antibiticos?*

El Dr. Alexander Fleming, quien observ que un moho Penicillium haba contaminado sus cultivos y de que algo estaba matando a las bacterias a su alrededor. Esta sustancia fue la penicilina, y 25 aos ms tarde, sirvi para salvar vidas en el campo de batalla (y tal vez algunos burdeles) en la Segunda Guerra Mundial. Se inicio la era de los antibiticos!

*Cul es la contribucin del Dr. Walter Reed a la medicina?*

* *El Dr. Reed instituyo el control del mosquito (vector) como un medio para controlar la fiebre amarilla, que diezm a las tropas durante la

Guerra Espaola-Americana y obstaculizado la conclusin del Canal de Panam y la expansin econmica de EE.UU. El control del mosquito ha erradicado la fiebre amarilla y la malaria en este continente y sigue siendo el principal medio de control (aunque tenga en cuenta que los mosquitos vectores estn siendo endmica a FL / SE Estados Unidos y si fueron patgenos para nuevamente iniciar la epidemia, el control del mosquitos sera la clave para el control y esos programas estn vigentes actualmente). El control de vectores es importante en el control de la malaria, encefalitis (como el del Nilo Occidental), el dengue, el virus hanta, la peste, etc

*Qu es el complejo de Ghon?*

Curada la infeccin tuberculosa primaria del lbulo inferior asociada con un nodulo hiliar calcificado. El sitio primario de infeccin por tuberculosis pulmonar.

Focalizacion de la infeccin tuberculosa primaria del lbulo inferior con asociacin de un nodulo hiliar calcificado. Sitio de la primoinfeccion tuberculosa.

*Cul es el enfoque de Simon?*

Infeccin de la TB lbulos superiores. Se trata de un sitio secundario de infeccin de la TB a travs de sangre o vasos linfticos del principal foco de Ghon.Puede causar cambios en el lbulo superior fibrosis antes de cualquier enfermedad activa. Este es el sitio de la reactivacin de costumbre, con neumona tpica de la tuberculosis, aos despus de la infeccin primaria.

*Cules son las formas de tuberculosis cutnea?*

la enfermedad de Bazin: papular, lesiones necrticas que representan las reacciones de hipersensibilidad a una mayor infeccin. Esp seal en la parte posterior de las piernas. Verruga prosector de se produce por inoculacin directa.

*Qu es la escrfula clsica?*

Cuello del tero y la tuberculosis faciales

*Cul es el origen de la hemoptisis masiva en la tuberculosis?*

aneurisma de Rasmussen es la erosin de un granuloma tuberculoso en una arteria pulmonar una muerte rpida.

*Describir las principales causas infecciosas y no infecciosas de la esplenomegalia.*

VEB / CMV Endocarditis TB Histoplasmosis Fiebre tifoidea La sfilis Parsitos Leishmaniasis (visceral) La malaria La esquistosomiasis (hipertensin portal no cirrtica) La toxoplasmosis Linfoma / dz mielodisplsicos Vascular del colgeno dz (AR, LES) Portal de la HTA (no cirrticos o con cirrosis) Anemias hemolticas hereditarias / policitemia vera La sarcoidosis Las metstasis Amiloide**

*Qu son las infecciones asociadas clsicamente con la patogenia?*

*B*azo *BELT*: *S*pleen-Salmonella/Staph aureus / estreptococo *B*-Brucelosis *E*-Endocarditis / E. coli *L-*enfermedad de la Lemierre (sepsis post-anginosa con Fusobacterium necrophorum) *T*-*T*B

In travelers from Thailand/SE Asia, think*melioidosis. *

En Tailandia, viajeros procedentes de Asia y el SE, que melioidosis.**

*Clsico escenario de absceso esplnico:*

trombocitosis inexplicable en un paciente en la UCI sptico con persistente derrame pleural izquierdo

*Enfermedades que se producen al mismo tiempo:*

La enfermedad de Lyme + Babesiosis + Ehrlichiosis (el vector es la garrapata) Sarampin + estreptococo Mono + estreptococo (faringitis mono puede imitar la garganta por

estreptococos, faringitis por estreptococo pueden ser super-infectar-el tratamiento de faringitis estreptoccica para prevenir la fiebre reumtica) Endocarditis + osteomielitis aguda

*6 Enfermedades de la Infancia:***

*El sarampin (Rubola)*

Prdromo tos, coriza, conjuntivitis, manchas de Koplik Rash eritematoso maculopapular, 5 das despus de la aparicin de la enfermedad, comienza en la cabeza y se extiende hacia abajo

*El sarampin alemn (Rubola)*

Los nios sin prdromos Adultos malestar general, fiebre, anorexia, adenopatas auricular posterior, cervicales y suboccipitales Rash maculopapular que comienza en la cara, luego se generaliza.

*Rosola infantil (Exantema sbito) del virus del herpes humano 6*

fiebre brusca, tiene una duracin de 1-5 das sin hallazgos fsicos. El cuarto da, erupcin macular o maculopapaular en el tronco, y se extiende perifricamente, resueltas en 24 horas. El nio luce generalment en buenas condiciones.

*Contra la varicela (varicela)*

Prdromo malestar general, fiebre, secrecin nasal. Erupcion Inicia el mismo da con fiebre y prurito, primero en el tronco, luego perifricamente; comienza como ppulas rojas, se convierte en lgrima vesculas, se nubla, se rompen, forma costras, se produce en cultivos. Recuerde: vesculas agrupadas sobre una base roja en las distintas etapas de la evolucin.

*El eritema infeccioso (quinta enfermedad)*

No tiene prdromo, 3 etapas de la erupcion pruriginosa: (1) Marcado eritema en las mejillas, mejilla abofeteada (2) livideses reticularis: comienza eritematosa, erupcin maculopapular en los brazos, luego el tronco y las piernas. (3) Dura 2-39 das, las fluctuaciones en la gravedad de la erupcin a los cambios ambientales. (4) artralgias principalmente en los adultos.

*Escarlatina*

Estreptococos del grupo A, fiebre, faringitis, erupcin Erupcion eritematosa (Rash eritematoso), finamente punteada, blanquea con la presin, se inicia en el tronco luego se generaliza. La cara est enrojecida, aumento de eritema en los pliegues cutneos (lneas de Pastia), la piel se siente spera como papel de lija, lengua de fresa.

*Cules son los dos patgenos ms comunes en el gnero Nocardia y droga de eleccin?*

Nocardia asteroides (pulmn o cerebro) TMP / SMX, imipenem + amikacina Nocardia brasiliensis (linfangitis / micetomas) TMP / SMX, resistente a imipenem

*Nombre de las enfermedades causadas por Listeria monocytogenes, las fuentes comunes, y drogas de eleccion.*

Gastroenteritis Meningoencefalitis Granulomatosis * infantisptica (aborto espontneo o muerte fetal debido a la difusin de Listeria; microabscesos generalizados/ granulomas en el hgado y en el bazo; abundantes bacterias en la tincin de Gram del meconio) La sepsis neonatal / meningitis *

* Infeccin transplacentaria de enteritis materna / bacteriemia.

De los productos lcteos sin pasteurizar, quesos blandos, carnes fras / perros calientes / calor embutidos-hasta que emitan vapor. Tto: ampicilina, TMP / SMX (Trimetropin sulfametoxasol)

You might also like